Sie sind auf Seite 1von 125

d. Provide him with continued 1. With R.A.

9155, to “You know, you should


understanding and support.
ANSWERS: which body were all the have not done that. Let’s
1. B functions, programs, and talk about it so you can
2. C
3. B activities of the handle the situation
4. B
5. C Department of Education better next time.”
6. B related to Sports Parent F – Tells her child:
7. C
8. B competition transferred? “You may do what you
9. A
10. B a. Technical Education want. We will always be
11. D
12. B Services Department here for you, no matter
13. C Authority what you do.”
14. A
15. A b. Philippine Sports Which Parenting style is
16. B
17. D Commission ***** Authoritarian?
18. A
19. C c. National Commission a. D b. F c.
20. A for Culture and the Arts E d. C*****
21. D
22. B d. Commission on Higher
23. D
24. C Education 3. Two identical beakers
25. B
26. C A and B are presented to
27. A
2. Parenting style the child. Teacher Sonny
28. B
29. B influences children’s pours the liquid from B to
30. B
31. D development. Read the C which is taller and
32. B
33. D following parent’s thinner than A and B but
34. A
35. C
remarks for their children has equal capacity with
36. C then, answer the B. The teacher asks if the
37. C
38. A question. beakers A and C have
39. D
40. C Parent C – Tells her the same amount of
41. A
42. A
child: “You should do it liquid. The child says
43. D my way or else. There is “NO” and points to C as
44. B
45. D no discussion.” the beaker that has more
46. B
47. C Parent D – Tells her liquid. In which cognitive
48. D
49. A
husband: “It is 10:00 PM, developmental stage is
50. D do you know where your the child?
child is?” a. Sensorimotor stage
Parent E – Tells her child: b. Concrete operational
stage 6. You arrange the rows asked the class to share
c. Pre-operational stage of blocks in such a way any insight derived from
***** that a row of 5 blocks is the poem. In which
d. Formal Operational longer than a row of 7 domain in Bloom’s
stage blocks. If you ask which taxonomy of objectives is
row has more, Grade 1 the term paraphrase?
4. To determine her pupils will say that it is a. Analysis c.
students’ level of moral the row that makes the Comprehension *****
development, Teacher longer line. Based on b. Application d.
Evangeline presents to Piaget’s cognitive Synthesis
her class a morally development theory, what 9. Which characterizes a
ambiguous situation and problem is illustrated? constructivist teaching-
asks them what they a. Assimilation problem learning process?
would do in such a b. Accommodation a. Conceptual
situation. On whose problem interrelatedness *****
theory is Teacher c. Conservation problem b. Multiple perspectives
Evangeline’s technique ***** c. Authentic assessment
based? d. Egocentrism problem d. Passive acceptance of
a. Bruner 7. According to R.A. information
b. Kohlberg ***** 9155, a school head has 10. On what theory is the
c. Freud two roles, namely sequencing of instruction
d. Piaget administrative manager anchored?
and ____. a. Gagne’s hierarchical
5. According to R.A. a. Health officer theory *****
9155, which among the b. Instructional leader b. B.F. Skinner’s operant
following is considered ***** conditioning theory
the “heart of the formal c. Facilitator c. Bandura’s social
education system”? d. Guidance counselor learning theory
a. The pupil 8. After reading and d. Thorndike’s law of
b. The teacher paraphrasing Robert effect
c. The classroom Frost’s Stopping by the 11. A common complaint
d. The school ***** Woods on Snowy of teachers about pupils
Evening, Teacher Marko is this: “You give them
assignment, the following c. Reversion your lesson clear and
day they come without b. Conservation d. specific
any. You teach them this Accommodation
today, asks them 15. Which is the ideal
tomorrow and they don’t 13. Based on Bandura’s stage of moral
know. It is as if there is theory, which conditions development? Stage of
nothing that you taught must be present for a _____.
them at all.” Based on the student to learn from a a. Social contract
theory of information model? b. Universal ethical
processing, what must I. Attention III. Motor principle *****
teachers do to counteract reproduction c. Law and order
pupil’s forgetting? II. Retention IV. d. Good boy/good girl
I. Punish every child who Motivation
can’t give correct a. I and II c. I, II, III and 16. Cristina’s family had a
answers to questions. IV***** family picture when she
II. Work for meaningful b. I, II and III d. III and IV was not yet born. Unable
learning by connecting to see herself in the
lesson to what pupils family picture, she cried
know. 14. According to despite her mother’s
III. Reward every child Tolman’s theory on explanation that she was
who remembers past purposive behaviorism, not yet born when the
lessons. learning is goal-directed. family picture was taken.
a. III only c. II and III What is its implication to What does Cristina’s
***** teaching? behavior show?
b. I and III d. II only a. Evaluate lessons a. Limited social cognition
based on your objective/s b. Egocentrism *****
12. When small children ***** c. Semi-logical reasoning
call all animals “dogs”, b. Set as many objectives d. Rigidity of thought
what process is as you can
illustrated, based on c. Stick to your 17. To help a student
Piaget’s cognitive objectives/s no matter learn to the optimum,
development theory? what happens Vygotsky advises us to
a. Assimilation ***** d. Make the objective/s of bridge the student’s
present skill level and the is/are at work? are in the psychomotor
desired skill level by a. Id c. Ego domain
______. b. Id and Superego
a. Challenging c. d. Superego ***** 21. Sassi, a Grade I pupil
Inspiring is asked, “Why do you
b. Scaffolding ***** 20. Here are comments pray everyday?” Sassi
d. Motivating from School Head answered, “Mommy said
Carmen regarding her so.” Based on Kohlberg’s
18. Based on Piaget’s observations on teacher’s theory, in which moral
theory, what should a practice in lesson development stage is
teacher provide in the planning: Sassi?
formal operational stage? The words “identify,” “tell” a. Pre-convention level
a. Stimulating and “enumerate” are *****
environment with ample overused. Many times b. Conventional level
objects to play with they make use of non- c. In between
b. Games and other behavioral terms. Often conventional and post-
physical activities to their lesson objectives do conventional levels
develop motor skills not include value d. In between pre- and
c. Activities for formation and inculcation. post-conventional levels
hypothesis formulation What can be inferred
***** from the School Head’s 22. Teacher Fatima tells
d. Learning activities that comments regarding her students: “You must
involve problems of teacher formulated be honest at all times not
classification and lesson objectives? only because you are
ordering a. Often lesson objectives afraid of the punishment
are in the low level but more because you
19. “Do not cheat. b. Very often lesson yourselves are convinced
Cheating does not pay. If objectives are in the of the value of honesty.”
you do, you cheat cognitive domain ***** Based on Kohlberg’s
yourself” says the c. Quite often lesson theory, which level of
voiceless voice from objectives describe moral development does
within you. In the context teacher’s behavior the teacher want her
of Freud’s theory, which d. Often lesson objectives students to reach?
a. Conventional level tasks because ______. b. Post-conventional
b. Between conventional a. Development is c. Conventional *****
and post-conventional affected by cultural d. Cannot be specifically
levels changes determined
c. Between pre- b. The development of
conventional and post- individuals is unique 27. In planning for
conventional levels c. Development is the instruction, can a teacher
d. Post-conventional individual’s choice begin with assessment?
level***** d. Development is aided a. No, it may discourage
by stimulation ***** and scare the learners
23. Why is babyhood b. Yes, determine entry
referred to as a “critical 25. According to knowledge or skill *****
period” in personality Havighurst’s c. Yes, to make the class
development? Because: development tasks, pay attention
a. At this time the baby is reaching and maintaining d. No, assessment is only
exposed to many satisfactory performance at the end of a lesson
physical and in one’s occupational
psychological hazards career is supposed to 28. Which among the
b. Changes in the have been attained following is closest to the
personality pattern take during ____. real human digestive
place a. Middle age and Early system for study in the
c. At this time the adulthood classroom?
foundations are laid upon b. Middle age ***** a. Drawing of the human
which the adult c. Old age digestive system on the
personality structure will d. Early adulthood board
be built ***** b. Model of the human
d. The brain grows and 26. Student Deina says: digestive system *****
develops at such an “I have to go to school on c. The human digestive
accelerated rate during time. This is what the rule system projected on an
babyhood says.” In what level of OHP
moral development is the d. Drawing of the human
24. It is good to give student? digestive system on a
students creative learning a. Pre-conventional page of a textbook
terms, this is how the d. Small group discussion
29. Here is a question: “Is teacher developed the
the paragraph a good lesson. 34. I want to use a pre-
one?” Evaluate. If broken Did the lesson begin with teaching strategy that will
down to simplify, which is concrete experience then immediately engage my
the best simplification? developed into the students in the content
a. Why is the paragraph a abstract? and will enable me to get
good one? Prove a. No ***** an insight into how
b. Is the paragraph a b. Yes, a little students think and feel
good one? Why or Why c. Yes, by way of the about the topic. Which is
not? ***** examples given by the most appropriate?
c. If you asked to teacher a. K-W-L chart ***** c.
evaluate something, what d. Yes, the pupils were Graphic organizer
do you do? Evaluate the involved in arriving at the b. Story boarding
paragraph? rule on reducing fractions d. Document analysis
d. What are the qualities to their lowest terms 35. For a discussion of a
of a good paragraph? 32. I want to compare two topic from various
Does the paragraph have concepts. Which perspectives, it is best to
these qualities? technique is most hold a ______.
appropriate? a. Debate c. Panel
30. Which one is in a. Attribute wheel discussion *****
support of greater b. K-W-L techniques b. Brainstorming d.
interaction? c. Venn diagram ***** Symposium
a. Probing ***** d. Spider web organizer 36. After establishing my
b. Repeating the question 33. Which activity should learning objectives, what
c. Not allowing a student a teacher have more for should I do to find out
to complete a response his students if he wants what my students already
d. Selecting the same them to develop logical- know and what they do
student respondents mathematical thinking? not yet know in relation to
a. Focus group my lesson objectives in
31. With this specific discussion the cognitive domain?
objective, to reduce b. Problem solving ***** a. Give a pretest *****
fractions to their lowest c. Games b. Study the least learned
competencies in the a. Allegorical thinking realias all the time?
National Achievement b. Concrete thinking a. No, for the sake of
Test c. Metaphorical thinking variety of instructional
c. Analyze my students’ ***** materials *****
grades last year d. Symbolical thinking b. No, only when feasible
d. Interview a sample of 40. Which must be c. Yes, because there is
my students primarily considered in no substitute for realias
37. What characterizes the choice of instructional d. Yes, because it is the
genuine change? Change aide? real thing
in _____. a. Must stimulate and 43. I want my students to
a. Appearance c. maintain student interest look at the issues on the
Substance ***** b. Must be updated and call for President Arroyo
b. Form d. Physical relevant to Filipino setting to step down from several
attribute c. Must be suited to the perspectives. Which
38. In which strategy, can lesson objective ***** activity is most fitting?
students acquire d. Must be new and a. Cross examination
information from various skillfully made c. Symposium
perspectives, and led to 41. For lesson clarity and b. Panel discussion *****
reflective thinking and effective retention, which d. Debate
group consensus? should a teacher 44. I intended to inculcate
a. Debate observe, according to in my students the value
b. Small group discussion Bruner’s theory? of order and cleanliness. I
***** a. Begin teaching at the begin my lesson by
c. Panel discussion concrete level but go asking them to share
d. Symposium beyond it by reaching the their experiences about
39. At the end of my abstract ***** the dirtiest and the
lesson on the role of a b. Use purely verbal cleanest place they have
teacher in learning, I symbols in teaching seen and how they felt
asked the class: “In what c. Start at the concrete about them. From there I
way is a teacher an level and end there lead them to the
enzyme?” With this d. End teaching with consequences of dirty
question, it engaged the verbal symbols and clean home of
class in _______. 42. Is it advisable to use surroundings. In my
lesson development plan, follow the fashion or else d. To compare her
how do I proceed? students won’t listen to students to national
a. Transductively you norms *****
c. Deductively b. Your physical 49. Other than finding out
b. Inductively ***** d. appearance and voice how well the course
Concretely should be such that competencies were met,
students are helped to Teacher Kathy also
45. Teacher Neri wants to learn ***** wants to know her
develop the ability of c. Make good use of the students’ performance
sound judgment in his radio and TV in the when compared with
students. Which of the classroom other students in the
following questions d. Include singing in your country. What is Teacher
should he ask? teaching method Kathy interested to do?
a. What is the essayist a. Formative evaluation
saying about judging 47. I used the gumamela b. Authentic evaluation
other people? flower, a complete flower, c. Norm-referenced
b. With the elements of a to teach the parts of a evaluation *****
good paragraph in mind, flower. Which method did d. Criterion-referenced
which one is best written? I use? evaluation
***** a. Demonstration method 50. I want to help my
c. Why is there so much b. Type-study method students retain new
poverty in a country ***** information. Which one
where there is plenty of c. Drill method will I use?
natural resources? d. Laboratory method a. Questions c. Games
d. Of the characters in b. Mnemonics ***** d.
the story, with whom do 48. A teacher would use Simulations
you identify yourself? a standardized test 51. I want to use a
______. diagram to compare the
46. The teacher is the a. To serve as a unit test traditional and authentic
first audio-visual aid in b. To serve as a final modes of assessment.
the classroom. What examination Which one is most fit?
does this imply? c. To engage in easy a. Affinity diagram
a. You take care that you scoring b. Tree diagram
c. Venn diagram ***** d. Four construction, what does
d. Fishbone diagram 54. When I teach skills TOS mean?
that are critical to the a. Table of Specification
52. A big story in your learning of the next *****
local newspaper. You topics, what should I b. Team of Specifications
want to use the headlines employ? c. Table of Specifics
as an inquiry device. To a. Direct instruction d. Terms of
increase student b. Mastery learning ***** Specifications
participation, you might c. Socratic method 58. If I favor “assessment
begin by ____. d. Cooperative learning for learning,” which will I
a. Asking one to read the 55. I want my students to do most likely?
news story and interpret have mastery learning of I. Conduct a pre-test
what he read after a basic topic. Which can results
b. Asking the class to help? II. Teach based on pre-
infer connotations and a. Drill ***** test results
denotations from the b. Socratic method and III. Give specific feedback
headline ***** drill to students
c. Explaining what you c. Morrisonian technique IV. Conduct peer tutoring
believe to be the and drill for students in need of
underlying causes d. Socratic method help
d. Describing the 56. Teacher Feng wanted a. I, II and IV c. I, II and
background of the story to teach the pupils the III
as you know it skill of cross stretching. b. II, III and IV d. I, II, III
53. If a triadic interaction Her check-up quiz was a and IV *****
includes three (3) written test on the steps 59. After a lesson on the
persons, how many of cross stitching. What atom, the students were
persons are included in a characteristic of a good asked to work on a
dyadic interaction? test does it lack? physical model of the
a. Two ***** a. Predictive validity atom to determine
b. Two groups composed b. Objectivity learning. For which group
of two persons each c. Reliability ***** of students is building an
c. One, the person and d. Content validity atom model intended?
himself 57. In the parlance of test a. Interpersonality
intelligent _______.
b. Kinesthetically 62. I want to present the a. Analyzing c.
intelligent ***** characteristics features of Synthesizing
c. Mathematically a constructivist approach. b. Inferring *****
intelligent What should I use? d. Generalizing
d. Linguistically intelligent a. Fishbone diagram 65. Which must go with
60. If I want to develop b. Venn diagram self-assessment for it to
creative thinking in my c. Narrative frame be effective?
students, which one/s d. Attribute wheel ***** a. Scoring rubric *****
should I use? 63. If all of your students b. Consensus of
I. Problem solving in your class passed the evaluation results from
II. Brainstorming pretest, what should you teacher and student
III. Dramatics do? c. External monitor
a. I and II c. III only a. Administer the posttest d. Public display of
b. II only d. I, II and b. Go through the unit as results of self-evaluation
II***** usual because it is part of 66. Which group of
the syllabus***** scores is most varied?
61. Which is/are effective c. Go through the lesson The group with
method/s in teaching quickly in order not to ________.
students critical reading skip any a. 0.90 SD ***** c. 0.10
skills? d. Go on to the next unit SD
a. Read and interpret 64. Teacher Vicky shows b. 0.50 SD d. 0.75 SD
three different movie her students a picture of 67. The main purpose in
reviews people in thick jackets. administering a pretest
b. Interpret editorials Then she asks them to and a posttest to students
about a particular subject tell her the kind of climate is to _____.
from three different when the picture was a. Measure gains in
newspaper ***** taken. If she asks 5 learning *****
c. Distinguish fiction from questions of this kind and b. Measure the value of
non-fiction materials her students do not get the material taught
d. Interpret editorials and them, it is safe to c. Keep adequate records
read and interpret three conclude that pupils are d. Accustom the students
different movie reviews quite weak in the skill of to frequent testing
68. Assessment is said to following groups? 74. A number of test
be authentic when the a. Above average items in a test are said to
teacher ________. b. Average***** be non-discriminating?
a. Consider students’ c. Below average What conclusion/s can be
suggestion in teaching d. Needs improvement drawn?
b. Gives valid and reliable 71. Which is true of a I. Teaching or learning
paper-and-pencil test bimodal distribution? was very good.
c. Gives students real-life a. The scores are neither II. The item is so easy
task to accomplish ***** high nor low that anyone could get it
d. Includes parents in the b. The group has two right.
determination of different groups***** III. The item is so difficult
assessment procedures c. The score are high that nobody could get it.
69. The following are d. The score are low a. II only c. III only
computed means of a 72. When you reach the b. I and II d. II and III*****
hundred-item test: “plateau of learning”, 75. A test item has a
Physical science, 38; what should you do? difficulty index of 0.51
Math, 52; English, 33. a. Forget about learning and a discrimination
Based on the data, which b. Reflect what caused it index of 0.25. What
is true? ***** should the teacher do?
a. The examinees seem c. Force yourself to learn a. Revise the item *****
to be very good in d. Rest b. Retain the item
Physical Science 73. What can be said if c. Make it a bonus item
b. The Math test appears student performance in a d. Reject the item
to be the easiest among positively skewed scores 76. The difficulty index of
the three ***** distribution? a test item is 1. This
c. The examinees seem a. Most students means that
to excel in English performed well _____________.
d. The English test b. Most students a. The test item is a
appears to be the easiest performed poorly***** quality item
among the three c. Almost all students had b. The test is very difficult
70. An examinee whose averaged performance c. The test is very easy
score is within x + 1 SD d. A few students *****
belongs to which of the performed excellently d. Nobody got the item
correctly discriminate between the d. Sixty-two percent
77. If the compound lower and upper groups (62%) of those who took
range is low, this means d. More from the upper the test scored higher
that _____________. group got the item than the individual
a. The students correctly 82. What does the
performed very well in the 80. In an entrance computer have in
test examination, student common with the TV?
b. The difference Anna’s Percentile is 25 a. Key board c.
between the highest and (P25). Based on this Screen*****
the lowest score is Percentile rank, which is b. File d. Disk drive
low***** likely to happen? 83. Which depicts in
c. The difference a. Student Anna will be graphic form the social
between the highest and admitted relations present in a
the lowest score is high b. Student Anna will not group?
d. The students be admitted***** a. Interest inventory
performed very poorly in c. Student Anna has 50- b. Sociogram*****
the test 50 percent chance to be c. Anecdotal record
78. What is the mastery admitted d. Johari’s window
level of a school division d. Student Anna has 75 84. Planned ignoring,
in a 100-item test with a percent chance to be signal interference, and
mean of 55? admitted proximity control are
a. 42% b. 55%***** 81. What does a techniques used in
c. 45% d. 50% percentile rank of 62 _____.
79. A negative mean? a. Managing temper
discrimination index a. It is the student’s score tantrums
means that in the test b. Managing surface
____________. b. The student answered behavior *****
a. The test item has low sixty-two (62%) of the c. Operant conditioning
reliability items correctly d. Life-space interviewing
b. More from the lower c. The student’s score is 85. What should you do
group answered the test higher than 62 percent of to get the child’s attention
item correctly***** all students who took the when she/he is distracted
c. The test item could not test***** by an object in the room?
I. Call him by his name Upon what philosophy is
and tell him to pay this anchored? 91. A wife who loves her
attention a. Realism c. husband dearly becomes
II. Put away the Idealism***** so jealous that in a
distracting influence b. Existentialism d. moment of savage rage,
III. Involve him in helping Pragmatism kills him. Is the wife
with an activity 88. In the schools, we morally responsible and
a. I and II ***** c. II and III teach realities that cannot why?
b. I and III d. I, II and III be verified by the senses a. Not necessarily.
86. Which practice in our like an Invisible God or Antecedent passion may
present educational Allah. Whose beliefs completely destroy
system is in line with does this practice freedom and
Plato’s thought that negate? consequently moral
“nothing learned under a. Stoicists’ c. Skeptics’ responsibility
compulsion stays with the b. Rationalists’ d. b. Yes, she killed her
mind”? Empiricists’***** husband simply because
a. Clarification of school 89. Which emphasized of jealousy*****
policies and classroom on non-violence as the c. It depends on the case
rules on Day 1 path to true peace as of the wife’s jealousy
b. Presentation of discussed in peace d. It depends on the
standards of performance education? mental health of the wife
in the learner a. Taoism c. Hinduism
c. Making the teaching- b. Buddhism d. 92. Martin Luther King Jr.
learning process Shintoism once said, “Man may
interesting***** 90. I make full use of the understand all about the
d. Involving the learner in question-and-answer as rotation of the earth but
the determination of a model for discussion. he may still miss the
learning goals From whom is this radiance of the sunset.”
87. In Values Education, question-and-answer Which type of education
faith, hope, and love are method? should be emphasized as
believed to be permanent a. Socrates ***** c. implied by Martin Luther
values whether they be Aristotle King Jr.?
valued by people or not. b. Kant d. Plato a. Science and education
b. Vocational education d. By merit system 99. Is a sick teacher, the
c. Liberal education***** 96. “Moral example has a only one assigned in a
d. Technical education greater effect on pupils’ remote school, excused
discipline than laws and from her teaching duty?
93. Computer-assisted codes of punishment” is a. No, she is the only one
instruction is an offshoot an advice of teachers assigned in that school
of the theory of _____. from _________. b. Yes, teaching is a
a. J. Piaget c. B.F. a. onfucius ***** c. Lao demanding job
Skinner***** tzu c. Yes, she is sick*****
b. J. Brunner d. J. b. Mohammed d. d. No, she must abide by
Watson Mencius her contract
94. The use of varied 97. “The principle of 100. What is a
teaching and testing spontaneity against demonstration of your
strategies on account of artificiality will make you authority as a
students’ multiple accomplish something. professional teacher?
intelligences is in line with Leave nature to itself and a. You make your pupils
the thoughts of ______. you will have harmony” is run errands for you
a. Daniel Goleman an active advice from the b. You decide on how to
b. Jean Piaget _____¬. teach a particular
c. Howard Gardner***** a. Hindu c. Shintoist lesson*****
d. Benjamin Bloom b. Taoist ***** d. c. You absent from
Buddhist class to enjoy your leave
95. Applying Confucius’s 98. The significance of even without prior notice
teachings, how would liberal education in d. You select to teach
hiring personnel select holistic development of only those lessons which
the most fit in students is supported by you have mastered
government positions? _____. 101. Which statement on
a. By record evaluation a. Perennialism ***** true authority is wrong?
done by an accrediting b. Pragmatism a. It sets an example
body c. Confucian teaching b. It seeks its own
b. By government d. Perennialism and satisfaction and
examinations***** Confucian philosophy privilege*****
c. By accreditation c. It acts in the best
interest of others b. No, non-attendance to 105. Is it ethical on the
d. Its goal is to help, form, seminars means no part of the teacher to
and guide others professional growth proselyte in her
102. When a teacher c. Yes, because she has classroom every Friday?
teaches the idea that it is taught for forty years and a. Yes, that strengthens
wrong to think that may have mastered the values education
Filipino lifestyle, products trade b. Yes, that is religious
and ideas are inferior to d. Yes, provided she has instruction which is
those of other an excellent performance allowed by the
nationalities, he fights rating Constitution
against ______. c. No, a teacher shall
a. Acculturation 104. A principal asked not engage in the
c. Ethnocentrism her good teachers to promotion of his/her
b. Xenocentrism ***** write modular lessons in religious interest in the
d. Culture shock Science, and then she classroom*****
103. Teacher Lolita, a had them published with d. No, proselyting is no
teacher for forty years, her name printed as longer necessary in this
refuses to attend author. Which is unethical age
seminars. She claims that in this case?
her forty years of a. She was the exclusive 106. Which can promote
teaching is more than all beneficiary of the royalty national pride among
the seminars she is from the modules pupils/students?
asked to attend. Is her b. She burdened her I. Studying the lives of
actuation and thinking in teachers with work not outstanding Filipinos here
accordance with the related to teaching and abroad
Code of Ethics for c. She had the modular II. Reading the lives of
Professional Teachers? lessons published when saints of the Church
a. No, a professional they were worth III. Studying Philippine
teacher, regardless of publishing history with emphasis on
teaching experience, d. She got the merit the victories and
ought to go through which was due for her greatness of the Filipino
continuing professional teacher-writers***** people
education***** a. I, II, III c. I & II
b. III only***** d. I only letter to prevent yourself pre-school
107. Why is the exodus from being involved 110. My right ends where
of Filipino professionals c. Talk to the married the rights of others begin.
described as “brain man with whom she is What does this mean?
drain”? having illicit affair a. Rights are not absolute
I. Those who go abroad d. Secretly give the *****
are usually the better anonymous letter only to b. Rights are alienable
II. Filipino professionals the two people concerned c. Rights are inalienable
serve other countries 109. Teachers often d. Rights are absolute
instead complain of numerous 111. History books used
III. They contribute to non-teaching in the schools are replete
nation building through assignments that with events portraying
their dollar remittances adversely affect their defeats and weaknesses
a. I, II, III b. I only c. II teaching. Does this mean of the Filipino people. In
only d. I & II***** that teachers must be the spirit of nationalism,
108. You want to report pre-occupied only with how should you tackle
on a colleague's act of their teaching? them in the classroom?
immorality. You don't a. Yes, because teaching a. Present them as they
have the courage to is enough full time job are and tell the class to
confront her. To end her b. Yes, if they are given accept reality
illicit affair with a married other assignments, b. Present the facts and
man you write and justice demands that they use them as means to
secretly distribute copies be properly teach and inspire your
of your anonymous letter compensated***** class*****
against your fellow c. No, because every c. Present the facts and
teacher. What should teacher is expected to express your feelings of
have been done instead? provide leadership in regret
a. If the charge is valid; activities for the d. Present the facts
present such charge betterment of the including those people
under oath before your communities where they responsible for the
school head***** live and work failures or for those who
b. Ask a third party to d. No, they are also baby contributed
write the anonymous sitters especially in the 112. Should an
association of teachers problem with another CASE #1 – Mrs. Domingo
obey a Supreme Court’s teacher, the first step developed a lesson on
decision even if it conflict towards resolution should the concept of fraction
with its interest and be for you to: this way: First, she
opinions? a. Talk directly with the presented one pizza, and
a. No teacher involved***** then asked a pupil to cut
b. Yes***** b. Ask your fellow it into two. She called one
c. Yes, provided they can teachers to intercede on part of the pizza 1/2 and
make a bargain your behalf the two parts of 2/2. Then
d. No, as a minority group c. Ask your fellow she wrote 1/2 and 2/2 on
they have the right to teachers for their the board. She
express their rejection suggestions proceeded to ask another
113. A teacher does not d. Discuss it with your pupil to divide the half
agree with the selective principal parts of the cake into two
retention policy of the 115. What is ethical for again, and then wrote
school and she openly you to do if deep within 1/2, 2/4 and 3/4. Then
talks against it in her your heart you do not she used the model of
classes. Is her behavior agree with the school fractions (wooden circles)
ethical? policy on student divided into 2, 3 and 4
a. Yes, provided she got absences? show 1/2, 1/4, 1/3, 2/4.
the permission from her a. Be vocal about your Then she went back to
superior to talk against feeling and opinion the fractions she wrote on
the policy against the policy the board. She asked her
b. No, it is her duty to b. Understand the policy pupils for the meaning of
faithfully carry it out even and support the school in 1/2, 1/3, 1/4, 2/4.
if she does not agree***** its implementation***** 116. Did Mrs. Domingo
c. No, in fact she is quite c. Argue with your follow Bruner's three
confused and passes on superior to convince him stage development of
her confusion to others to change the policy knowledge?
d. Yes, she is entitled to d. Keep your feeling to a. Yes*****
her opinion just as yourself but make b. No
everybody is insinuations that you are c. Only in the first stage
114. If you have a against it while you teach d. Cannot be determined
because the lesson was teacher their primary customer.
not developed fully d. Yes, provided we What follow is the gist of
117. In the context of include a concrete what were shared:
Bruner's principle of application of the abstract Teacher A - “The learner
knowledge 119. Which part of the is a product of his
representation, which is lesson is the symbolic environment. Sometimes
the enactive phase of the stage? he has no choice. He is
lesson on fractions? a. Using the model of determined by his
a. Presenting the pizza fractions environment.”
and cutting it into two and b. Dividing the pizza Teacher B - “The learner
four***** further into four can choose what he can
b. Using the model of c. Dividing the pizza into become despite his
fractions two environment.”
c. Writing the fractions d. Writing the fractions Teacher C - “The learner
1/2, 1/4 and 1/3 on the 1/2, 1/4, 1/3, 2/4 on the is a social being who
board board***** learns well through an
d. Asking the meaning of 120. Does the active interplay with
1/2, 1/4 and 1/3 development of the other.”
118. Would it be easier to lesson on fraction Teacher D - “The learner
understand and retain the conform to the bottom-up is a rational being.
concept of fractions if arrangement of the Schools should develop
Mrs. Domingo began the learning experiences in his rational and moral
lesson on fractions with Edgar Dale's Cone of powers.”
the meaning of 1/2, 1/3, Experience? 121. Whose philosophical
1/4, etc.? a. No concept is that of
a. Yes, provided we b. Cannot be determined Teacher A?
proceed to the concrete c. Yes***** a. Rationalist c.
b. No, for better learning d. Up to the second Existentialist
we proceed from the phase of the lesson only b. Behaviorist***** d.
concrete to the CASE #2 – In a faculty Progressivist
abstract***** recollection, the teachers 122. Teacher B's
c. It depends on the were asked to share their response comes from the
teaching skills of the thoughts of the learner, mouth of a/an:
a. Behaviorist c. tested principles b. Make their students
Essentialist b. Hypotheses***** d. derive meaning from
b. Existentialist***** d. Laws what is presented
Perennialist 127. Which material will c. Let their students
123. If you agree with her teachers most likely construct meaningful
Teacher C, you are more avoid? sentences based on the
of a/an: a. Unquestionable lesson*****
a. Perennialist c. laws***** d. Give the meaning of
Essentialist b. Open-ended topics what they present in
b. Rationalist d. c. Problems or cases class
Progressivist***** d. Controversial issues 130. Which one should
124. If you identify 128. On which her teachers then avoid?
yourself with Teacher D, assumption/s is/are the Students':
you adhere to what principal's action a. Reflection
philosophy? anchored? b. Self-directed learning
a. Progressivist I. Students learn by c. Memorization of facts
c. Existentialist personally constructing for testing*****
b. Behaviorist d. meaning of what is taught d. Inquiry
Perennialist***** II. Students construct and
125. Whose response reconstruct meanings CASE #4 – On the first
denies man's free will? based on experiences day of school, Teacher
a. Teacher A***** c. III. Students derive Yveta oriented her class
Teacher D meaning from the on procedures to be
b. Teacher C d. Teacher meaning that teacher observed in passing
B gives papers, getting textbooks
CASE #3 – School Head a. I only c. I and II***** from and returning the
Amilia wants her teachers b. I and III d. II only same on the bookshelf,
to be constructivist in 129. Which does School leaving the room for
their teaching orientation. Head Amilia want her necessity, and
126. Which material will teachers to do? conducting group work.
her teachers most likely a. Require their students She arranged the chairs
use? to come up with a in such a way that
a. Facts c. Time- construct of the lesson students can interact and
can move around without Democratic***** boring and will work when
unnecessarily distracting b. Authoritarian d. imposed
those seated. She Laissez-faire b. Classroom routines
involved the class in the 133. Which adjective leave more time for class
formulation of rules to appropriately describes instruction*****
ensure punctuality, order Teacher Yveta as a c. Students need to be
and cleanliness in the classroom manager? treated like adults to learn
classroom. a. Proactive ***** c. responsibility
131. On what belief is Reactive d. Teacher's personality
Teacher Yveta's b. Modern d. Traditional is a critical factor in
management practice 134. Was it sound classroom discipline
anchored? classroom management
a. Classroom rules need practice for Teacher CASE #5 – Mr. Santo's
to be imposed for order's Yveta to have involved lesson was on water
sake the students in the conservation. He
b. The classroom formulation of class presented a graph that
environment affects rules? compared water
learning***** a. No, it weakens the consumption of small and
c. A teacher must lord teacher's authority over big families. Before he
her power over her her students asked any of the
students to be an b. Yes, it makes students questions, he asked
effective classroom feel a sense of ownership someone to stand up to
manager of the rules***** give an answer. He called
d. A reactive classroom c. Yes, it lessens a only on those who raised
management style is teacher's work their hands. The
effective d. No, it is a students' act questions he asked were:
132. Teacher Yveta of usurpation of teacher’s 1. What do you see in the
involved her students in power graph?
the formulation of class 135. Which assumption 2. How do you compare
rules. Which describes underlies Teacher the water consumption of
her classroom Yveta's classroom small and big families?
management style? management practice? 3. Why do most of the big
a. Benevolent c. a. Classroom routines are families consume more
water than the smaller interaction? observations, in what
families? I. Asking high-level ways for people
4. Do all the small questions contribute to water
families consume less II. Calling only on those wastage?
water than the big who raised their hands c. What maybe the
families? Explain your III. Calling on someone reason why even if
answer. before asking the Family B is not as big as
5. In your opinion, why question Family C, it consumes
does one small family a. II and III***** c. III only much more water than
consume more water b. I and II d. I and III Family C? *****
than one big family? 138. Which statement on d. Among the families,
6. In what ways is water “wait time” is correct? who contributes most to
wasted? a. The higher the level of water conservation?
7. What are ways of the question, the longer 140. Were all the
conserving water? the wait time questions of Mr. Santo
8. Are the families b. Wait time turns off slow divergent?
presented well at water thinking students***** a. Yes c. No*****
conservation? Why or c. For quality response, b. No, except #4
why not? “what” and “why” d. Yes, except #1
9. What generalization questions require equal
can you draw about water wait time CASE #6 – With a topic
consumption and size of d. Wait time discourages on the human circulatory
families? the brighter group of system, Teacher Jan
136. Is there any students formulated the following
convergent question from 139. To connect the lesson objectives:
#1 to #8? lesson on water 1. Given a model of the
a. Yes, question #4 conservation to the life of human circulatory
b. Yes, question #7 ***** the students, which system, the student must
c. Yes, question #8 question is most be able to understand the
d. None appropriate? route of blood circulation
137. Which question a. How can you help 2. After discussing the
technique/s of Mr. Santo conserve water? process of blood
do(es) not enhance b. Based on you circulation, the teacher
must be able to lead the lesson objective cited by of the lesson after a usual
pupils in enumerating Robert Mager? forty-five minute lecture,
circulatory system-related a. Only objective #1 has she finds out that only
diseases and in citing the b. Only objective #2 has one or two can answer
causes and prevention of c. No, they don't ***** her questions. This has
such diseases. d. Yes, they do become a pattern so one
141. Is objective #1 in 144. Is objective #2 in time, when the class
accordance with the accordance with the could not answer, she
principles of lesson principles of lesson gave a test.
objective formulation? objective formulation? 146. What does the
a. No, the word a. No, it does not inattentiveness of most of
“understand” is not a describe pupils' learning Teacher Fantina's
behavioral term***** behavior***** students confirm?
b. No, it is not attainable b. Yes, it is formulated a. The “ripple effect” of
c. Yes, “understand” is from the point of view of behavior *****
an action word the teacher b. The lack of academic
d. Yes, it is very specific c. No, it is very broad preparation of some
142. How can you d. Yes, it describes teachers
improve objective #2? teacher's teaching activity c. The strange
a. Remove the phrase 145. Which one is the behaviors of today's
“After discussing the measurable learning students
process of blood behavior in objective #1? d. The stubbornness of
circulation.” a. Able to understand student groups
b. Formulate it from the b. Route of blood
learner's point of circulation 147. Which method in
view***** c. To understand dealing with classroom
c. Cut it short; the d. None ***** management problem is
statement is quite long better than that of
d. No need to improve on CASE #7 – Teacher Teacher Fantina?
it Fantina has a hard time a. Low level force and
143. Do both objectives getting the attention of private communication
include a criterion of her class. When she b. Low level force and
success, an element of a checks for understanding public communication
c. High level force and of teaching? *****
private communication a. Asking questions to 153. Teacher Nikka
d. High level force and check for understanding wants to check prior
public b. Giving a lecture knowledge of her pupils
communication***** c. Checking for about water pollution.
148. Can the understanding of the She writes the main topic
inattentiveness of lesson in the process of water pollution in the
Teacher Fantina's class teaching center of the chalkboard
be attributed to her use of d. Giving a test to and encircles it. Then,
the lecture method? discipline the class***** she ask the pupils to
a. Yes, if the lecture was 151. Teacher Leon gives provide information that
not interactive***** his students opportunities can be clustered around
b. Yes, if Teacher Fantina to be creative because of the main topic. Which
is an experienced teacher his conviction that much technique did the teacher
c. No, if the students are learning results from the employ?
intelligent need to express a. Vocabulary building
d. Cannot be determined creativity. On which b. Semantic mapping
149. With the principles theory is Teacher Leon’s *****
of learning in mind, which conviction anchored? c. Demonstration
one can help Teacher ______ theory d. Deductive teaching
Fantina solve her student a. Behaviorist c. 154. The current
disciplinary problem? Cognitive emphasis on the
a. Call on their first b. Associationist d. development of critical
names Humanist ***** thinking by the use of
b. Do interactive teaching 152. Which philosophy philosophic methods that
***** approves of a teacher emphasize debate and
c. Change seat plan of who lectures most of the discussion began with:
the class time and requires his a. Aristotle c. Confucius
d. Assign monitors in students to memorize the b. Socrates ***** d. Plato
class rules of grammar? 155. Which refers to a
150. Which act of a. Existentialism c. single word or phrase
Teacher Fantina is Pragmatism that tells the computer to
contrary to the principles b. Realism d. Idealism do something with a
program or file? 5, 6, 7, 8, 9, 10? sectarian schools *****
a. Computer program c. a. 7.5 b. 8.5 b. Public schools
Computer language c. 6 d. 7***** c. Sectarian and non-
b. Password d. 160. Which is a teaching sectarian schools
Command ***** approach for d. Private schools
156. In instructional kindergartens that makes
planning, which among real world experiences of 163. Which among
these three; unit plan, the child the focal point of following can help
course plan, lesson plan educational stimulation? student development the
is most specific? a. Situation approach habit of critical thinking?
_________ plan. b. Traditional approach a. Blind obedience of
a. Course c. Resources c. Montessori approach authority
b. Unit d. Lesson ***** ***** b. A willingness to
157. The first American d. Eclectic approach suspend judgment until
teachers on the 161. Which among the sufficient evidence is
Philippines were: following graphic presented *****
a. Missionaries organizers used helps to c. Asking convergent
b. Soldiers***** show events in questions
c. Graduates of the chronological order? d. Asking low level
normal school a. Time line and story questions
d. Elementary graduates map
158. By which process do b. Time line and cycle 164. Teacher Nelda
children become c. Series of events chart wants to develop in her
participating and and story map pupils comprehension
functioning members of d. Time line and series of skills. What order of skills
society by fitting into an events chart ***** will she develop?
organized way to life? I. Literal comprehension
a. Socialization ***** c. 162. Which schools are II. Interpretation
Accommodation subject to supervision, III. Critical evaluation
b. Acculturation d. regulation and control by IV. Integration
Assimilation the state? a. II-III-IV-I c. I-II-III-
159. What is the mean of a. Public, private IV*****
this score distribution 4, sectarian and non- b. III-IV-I-II d. IV-III-II-I
165. An integrative, having students grade system to be set in the
conceptual approach papers Philippines
introduced by Roldan that d. Assigning exercises c. The first religious
has as its highest levels from a workbook group who came to the
in the development of 169. The process of task Philippines on board the
_____ thinking skills. analysis ends up in the US transports Thomas
a. Interpretative formulation of: d. The devotees to St.
c. Critical ***** a. Instructional Thomas Aquinas who
b. Creative d. Literal objectives***** came to evangelize
166. If a student thinks b. Enabling objectives 172. Which teaching
about thinking, he is c. Goals of learning activity is founded on
involved in the process d. Behavioral objectives Bandura's social learning
called ________. theory?
a. Higher order thinking 170. For city-bred a. Questioning
b. Metacognition ***** students to think that their b. Inductive reasoning
c. Critical thinking culture is better than c. Modeling *****
d. Creative thinking those from the province is d. Interactive teaching
167. The use of drills in a concrete example of 173. Which program is
the classroom is rooted ____. DepEd’s vehicle in
on Thorndike’s law of: a. Ethical relativism mobilizing support from
a. Readiness c. c. Cultural relativism the private and non-
Exercise b. Ethnocentrism***** d. government sectors to
b. Effect d. Xenocentrism support programs based
Belongingness 171. Who were the on DepEd’s menu of
168. The following are Thomasites? assistance packages?
some drill techniques, a. The first American a. Chili-Friendly-School
except: teachers that help in System
a. Challenging students establishing the public b. Adopt-A-School-
to be above the level of educational system in the Program *****
the class Philippines ***** c. Every Child A Reader
b. Asking pupils to repeat b. The soldiers who Program
answers ***** doubted the success of d. Brigada Eskwela
c. Giving short quiz and the public educational 174. Researchers gave
rats a dose of 3-m butyl technical subjects c. Are the experiences in
phthalide and measured d. Set aside the study of accordance with the
changes in the rats blood local history patterns of pupils?
pressure. This statement d. Do the experiences
is best classified as 177. Which is in encourage pupils to
a. Experiment***** c. accordance with the inquire further?
Hypothesis “with-it-ness” principle of 179. Which is a
b. Prediction d. Finding classroom management characteristic of an
of Kounin? imperfect type of
175. The right a. Students agree to matching test?
hemisphere of the brain disagree in class a. An item may have no
is involved with the discussions answer at all *****
following functions b. Teacher is fully aware b. An answer may be
except: of what is happening in repeated
a. Visual functions his classroom***** c. There are two or more
b. Nonverbal functions c. Student is with his distracters
***** teacher in everything he d. The items in the right
c. Intuitive functions teaches and left columns are
d. Detail-oriented d. Both parents and equal in number
functions teachers are involved in 180. Which is a proactive
176. To build a sense of the education of children management practice?
pride among Filipino a. Tell them that you
youth, which should be 178. Below are questions enforce the rules on
done? that must be considered everyone, no exception
a. Re-study our history in developing appropriate b. Set and clarify your
from the perspective of learning activity rules and expectation on
our colonizer experiences except one. Day 1*****
b. Re-study our history Which is it? c. Punish the
and stress on our a. Can experiences misbehaving pupils in the
achievements as a benefit the pupils? presence of their
people ***** b. Do the experiences classmates
c. Replace the study of save the pupils from d. Stress on penalty for
folklores and myths with learning difficulties? ***** every violation
181. “Specialization is in the classroom for one 186. Which learning
knowing more and more reason or another, which principles is the essence
about less and less. Then of the following was of Gardner’s theory of
it is better to be a established? multiple intelligences?
generalist,” claims a. Informal a. Almost all learners are
Teacher Patty. On which b. Special education linguistically intelligent
philosophy does Teacher (SPED) b. Intelligence is not
Patty learn? c. Pre-school education measured on one form
a. Essentialism ***** c. d. Alternative learning *****
Perennialism delivery system ***** c. Learners have different
b. Progressivism d. 184. Tira enjoys games IQ level
Existentialism like scrabble, anagrams d. Learners have static IQ
182. I like to develop the and password. Which 187. Who asserted that
synthesizing skills of my type of intelligence is children must be given
students. Which one strong in Tira? the opportunity to explore
should I do? a. Interpersonal and work on different
a. Ask my students to intelligence materials so that they will
formulate a b. Linguistic intelligence develop the sense of
generalization from the ***** initiative instead of guilt?
data shown in the graphs c. Logical and a. Kohlberg c. Maslow
***** mathematical intelligence b. Erickson ***** d.
b. Direct my students to d. Spatial intelligence Gardner
point out which part of the 185. Teacher Milla 188. Teacher Ada uses
graph are right and which observes cleanliness and direct instruction strategy.
part is wrong order in the classroom to Which will she first do?
c. Ask my students to create a conducive a. Presenting and
answer the questions atmosphere for learning. structuring
beginning with “what if” On which theory is her b. Independent practice
d. Tell my students to practice based? c. Guided student
state data presented in a. Psychoanalysis practice
the graph b. Gestalt psychology d. Reviewing the previous
183. To reach out to c. Behaviorism ***** day’s work *****
clientele who cannot be d. Humanistic psychology 189. The free public
elementary and b. To support viewpoints specific questions to elicit
secondary educations in against abortion responses*****
the country are in the line c. To respond positively c. The students did not
with the government to a comment understand what you
effort to address d. To formulate criteria for were talking about
educational problems of honestly d. The students
_________. 192. “Using the six understood everything
a. Productivity descriptions of elements you presented
b. Relevance and quality of good short story, 194. Teacher Vina feels
c. Access and quality IDENTIFY IN WRITING offended by her
***** THE SHORT STORY BY supervisor’s unfavorable
d. Effectiveness and O. HENRY, with comments after a
efficiency complete accuracy.” The classroom supervision.
190. Here are raw scores words in capital letters She concludes that her
in a quiz 97, 95, 85, 83, are referred to as the supervisor does not like
77, 75, 50, 10, 5, 2, 1. To ____________. her. Which Filipino trait is
get a picture of the a. Criterion of success demonstrated by Teacher
group’s performance b. Condition Vina?
which measure of central c. Performance statement a. Extreme family-
tendency is most ***** centeredness
reliable? d. Minimum acceptable b. Extreme personalism
a. Median ***** performance *****
b. Mode c. “Kanya-kanya”
c. Mean 193. You have presented syndrome
d. None, it is best to look a lesson on animal d. Superficial religiously
at the individual scores protective coloration. At 195. Teachers are
the end, you ask if there encouraged to make use
191. Which objective in are any questions. There of authentic assessments
the affective domain is are none. You can take such as:
the lowest level? this to mean that ______. a. De-contextualized drills
a. To accumulate a. The students are not b. Unrealistic
examples of authenticity interested in the lesson performances
***** b. You need to ask c. Answering multiple
choices test items improve the above
d. Real world application 199. I’d like to test matching test?
of lessons learned ***** whether a student knows a. Add five items in both
196. What does the what a particular word columns
principle of individual means. What should I b. Add one or two items
difference require ask the student to do? in the right column *****
teachers to do? a. Give the word a tune c. Add ten items in both
a. Give less attention to then sing it columns to make the test
gifted learners b. Define the word ***** more comprehensive
b. Provide for a variety of c. Spell the word and d. Add one or two items
learning activities ***** identify its part of speech in the left column
c. Treat all learners alike d. Give the etymology of
while teaching the word 201. A teacher’s quarrel
d. Prepare modules for with a parent makes her
slow learners in class 200. Study this matching develop a feeling of
type of test. Then answer prejudice against the
197. The source of the question below. parent’s child. The
energy responsible for life 1. Measure of teacher’s unfavorable
on earth is the ______. relationship treatment of the child is
a. Moon c. Star 2. Measure of central an influence of what
b. Wind d. Sun ***** tendency Filipino trait?
198. Which quotation 3. Binet-Simon a. Lack of self-reflection
goes with a proactive 4. Statistical test of mean b. Extreme personalism
approach to discipline? difference *****
a. “An ounce of 5. Measure of variability c. Extreme family-
prevention is better than a. Mean centeredness
a pound of cure.” ***** b. Standard Deviation d. “Sakop-mentality”
b. “Do not make a c. Rho
mountain out of a d. T-ratio 202. In the context of
molehill.” e. Intelligence testing grading, what is referred
c. “Walk your talk.” movement to as teacher’s generosity
d. “Do not smile until Which among the error? A teacher
Christmas.” following is a way to _______.
a. Rewards students who c. The higher the the American regime
perform well standard deviation the were called Thomasite’s
b. In overgenerous with less spread the scores because:
praise are a. They were devotees of
c. Has a tendency to give d. It is a measure of St. Thomas Aquinas
high grades as compare central tendency b. They disembarked
to the rest ***** 205. What is the from the CIS Transport
d. Gives way to students’ Teacher’s called Thomas*****
bargain for no more quiz Professionalism Act? c. They first taught at the
203. For mastery learning a. RA 7836 ***** c. RA University of Sto. Tomas
and in line with outcome- 7722 d. They arrived in the
based evaluation model b. RA 4670 d. RA 9263 Philippines on the feast of
which element should be 206. The State shall St. Tomas
present? protect and promote the 208. Zazha exhibits fear
a. Inclusion of non- right of citizens to quality response to freely
performance objectives education at all levels. roaming dogs but does
b. Construction of Which government not show fear when a dog
criterion-referenced program is in support of is on a leash or confined
tests***** this? to a pen. Which
c. Construction of norm- a. Exclusion of children conditioning process is
referenced tests with special needs from illustrated?
d. Non-provision of the formal system a. Generalization c.
independent learning b. Free elementary and Discrimination*****
204. Which statement secondary education ***** b. Acquisition d.
about standard deviation c. Deregulated tuition fee Extinction
is correct? hike 209. The Filipino learner
a. The higher the d. Re-introduction of the envisioned by the
standard deviation the NEAT and NSAT Department of Education
more spread the scores 207. The American is one who is imbued with
are ***** Teachers who were the desirable values of
b. The lower the standard recruited to help set the person who is:
deviation the more public educational system a. Makabayan, makatao,
spread the scores are in the Philippines during makakalikasan at maka-
Diyos***** tells her teachers that on the community itself is
b. Makabayan, makatao, training in the humanities becoming a common
makahalaman, at maka- is most important. To phenomenon. What does
Diyos which education this signify?
c. Makabayan, philosophy does he a. Prevalence of poverty
makasarili, adhere? in the community
makakalikasan, at maka- a. Existentialism c. b. Inability of school to
Diyos Essentialism hire security guards
d. Makabayan, b. Progressivism d. c. Deprivation of Filipino
makakaragatan, Perennialism ***** schools
makatao, at maka-Diyos 213. Behavior followed by d. Community’s lack of
210. The right pleasant consequences sense of co-ownership
hemisphere of the brain will be strengthened and *****
is involved with the will be more likely to 215. The main purpose of
following function except: occur in the future. compulsory study of the
a. Intuitive functions Behavior followed by Constitution is to _____.
b. Nonverbal functions unpleasant a. Develop students into
***** consequences will be responsible, thinking
c. Visual functions weakened and will be citizen*****
d. Detail-oriented less likely to be repeated b. Acquaint students with
functions in the future. Which one the historical
211. What function is is explained? development of the
associated with the right a. Freud’s psychoanalytic Philippine Constitution
brain? theory c. Make constitutional
a. Visual, non-verbal, b. Thorndike’s law of experts of the students
logical effect***** d. Prepare students for
b. Visual, intuitive, non- c. B.F. Skinner’s Operant law-making
verbal conditioning theory 216. Which goals of
c. Visual, intuitive, logical d. Bandura’s social educational institution, as
***** learning theory provided for by the
d. Visual, logical, detail- 214. Theft of school Constitution, is the
oriented equipment like TV, development of work
212. Principal Connie computer, etc. by people skills aligned?
a. To develop moral good one? Why or why malaking titik? Malungkot
character not? ***** ANG MGA nagtapos na
b. To develop vocational b. Why is the paragraph a wala pang trabaho.
efficiency ***** good one? Prove a. Pananda ***** c.
c. To teach the duties of c. If you asked to Pariralang pantukoy
citizenship evaluate something, what b. Pang-ukol d.
d. To inculcate love of do you do? Evaluate the Pangatnig
country paragraph 222. Which is NOT a
217. Complete this d. What are the qualities characteristic of
analogy: of a good paragraph? education during the pre-
Spanish period: moral Does the paragraph have Spanish era?
and religious person. these qualities? a. Vocational training-
American period: 220. Manunulat ang tatay oriented
_______ mo kaya sa personal b. Structured *****
a. Productive citizen mong kagustuhan c. Unstructured
*****c. Patriotic citizen makasunod sa kanyang d. Informal
b. Self-reliant citizen mga yapak, magsusulat 223. Which is closest to
d. Caring citizen ka. Ano ang kahulugan the real human digestive
218. Who is remembered ng pagsusulat? system for study in the
for his famous quotation? a. Proseso na classroom?
“My loyalty to may party nagsisimula sa idea o a. Model of the human
ends where my loyalty to karanasan ***** digestive system *****
my country begins.” b. Makalikha ng mga b. Drawing of the human
a. Carlos P. Garcia c. salita, pangungusap at digestive system on the
Manuel L. Quezon ***** talata board
b. Ferdinand Marcos d. c. Kakayahan ng tao na c. The human digestive
Manuel Roxas makapagtala o system projected on an
219. Here is a question: makapagimprenta ng OHP
“Is the paragraph a good malinaw d. Drawing of human
one? Evaluate.” If broken d. Pagpili ng paksa at digestive system on a
down to simplify, which is pagsasaliksik page of a textbook
the best simplification? 221. Anong bahagi ng 224. Which one is in
a. Is the paragraph a pananalita ang nasa support of greater
interaction? 227. What is the mastery a. Socratic method and
a. Repeating the question level of school in a 100 drill
b. Not allowing a student item test with a mean of b. Drill *****
to complete a response 55? c. Socratic method
c. Probing ***** a. 42% b. 50% c. 45% d. None of the above
d. Selecting the same d. 55%***** 231. As a teacher you are
student respondents a skeptic. Which among
225. Which is/are 228. Who stressed the these will be your guiding
effective methods/s in idea that students cannot principle?
teaching students critical learn if their basic needs a. I must teach the child
reading skills? are not first met? to the fullest
a. Interpret editorials a. Thorndike b. I must teach the child
about a particular subject b. Maslow ***** every knowledge, skill,
from three different c. Wertheimer and value that he needs
newspapers d. Operant conditioning for a better future
b. Read and interpret c. I must teach the child
three different movie 229. A person, who has that we can never have
reviews had painful experience as real knowledge of
c. Distinguish fiction from the dentist’s office, may anything *****
non-fiction materials become fearful at the d. I must teach the child
d. Interpret editorials and mere sight of the dentist’s so he is assured
read and interpret three office building. What 232. With forms of
different movie reviews theory can explain this? prompting in mind, which
***** a. Attribution theory group is arranged from
226. Which is true of a b. Classical conditioning least to most instructive
bimodal score ***** prompting?
distribution? c. Generalization a. Verbal, physical,
a. The group tested has d. Operant conditioning gestural
two different groups ***** b. Verbal, gestural,
b. The scores are neither 230. I want my students physical *****
high nor low to have mastery learning c. Gestural, physical,
c. The scores are high of a basic topic. Which of verbal
d. The scores are low the following can help? d. Physical, gestural,
verbal d. Become aware of the c. Learning as it normally
233. In which way does pollutants in the occurs
heredity affect the environment d. Authentic problem
development of the 236. To encourage solving
learner? introspection, which 239. I want to engage my
a. By providing equal teaching method is students in small group
potential to all MOST appropriate? discussions. Which topic
b. By making acquired a. Cognitive c. lends itself to a lively
traits hereditary Process discussion?
c. By compensating for b. Reflective***** d. a. The exclusion of Pluto
what environment fails to Cooperative learning as a planet *****
develop 237. With indirect b. The meaning of the
d. By placing limits instruction in mind, which law of supply and
beyond which the learner does not belong to the demand
cannot develop ***** group? c. The law of inertia
234. In writing a. Lecture-recitation ***** d. Rules on subject-verb
performance objectives, b. Experiential method agreement
which word is not c. Inductive method 240. Teacher Joshua
acceptable? d. Discovery method discovered that his pupils
a. Integrate c. 238. A master teacher is are weak in
Manipulate the resource speaker in comprehension. To
b. Delineate d. an in-service training. He further in which particular
Comprehend ***** presented the situated skill(s) his pupils are
235. For which lesson learning theory and weak, which test should
objective will a teacher encouraged his Teacher Joshua give?
use the direct instruction colleagues to apply the a. Standardized test c.
method? same in class. Which of Placement
a. Distinguish war from the following did he not b. Aptitude test d.
aggression encourage his colleagues Diagnostic *****
b. Appreciate Milton’s to do? 241. The students of
Paradise Lost a. Apprenticeship***** Teacher Kath scan an
c. Use a microscope b. Decontextualized electronic encyclopedia,
properly ***** teaching view a film on the subject,
or look at related topics at III. You address the Divergent
the touch of a button right needs of the class as a 246. I want to teach
there in the classroom. whole within the context concepts, patterns, and
Which devices does of the learners with abstractions. Which
teacher Kath’s class specific learning or social method will be most
have? needs appropriate?
a. Videotape lesson c. a. II only c. I only a. Discovery c. Direct
Video disc b. I and II d. I and III***** instruction *****
b. Teaching 244. Which will be the b. Indirect instruction d.
machine***** d. CD most authentic Problem solving
242. If my approach to assessment tool for an 247. We are very much
my lesson is behaviorist, instructional objective on interested in a quality
what features will working with and relating professional development
dominate my lesson? to people? program for teachers.
I. Lecturing III. a. Organizing a What characteristic
Reasoning community project ***** should we look for?
II. Copying notes IV. b. Writing articles on a. Required for renewal
Demonstration working and relating to of professional license
a. III, IV c. I, II, III, IV people b. Prescribed by top
b. I, III, IV d. II, III, IV c. Home visitation educational leader
***** d. Conducting mock c. Responsive to
243. You practice election identified teacher’s needs
inclusive education. 245. Here is a test item: *****
Which of these applies to “From the data presented d. Dependent on the
you? in the table, form availability of funds
I. You accept every generalizations that are 248. What principle is
student as full and valued supported by the data.” violated by overusing the
member of the class and Under what type of chalkboards, as though it
school community question does this item is the only education
II. Your special attention fall? technology available?
is on learners with a. Convergent c. a. Isolated use c.
specific learning or social Application Variety *****
needs b. Evaluative ***** d. b. Flexibility d.
Uniformity materials, etc. Which of a. Low-level c. Analysis
249. Which statement the following did the b. Evaluative***** d.
applies correctly to Edgar school use? Convergent
Dale’s “CONE of a. Symposium c. Role 253. Teacher Jenny
experience”? playing teaches a lesson in which
a. The farther you are b. Simulation ***** students must recognize
from the base, the more d. Philips 66 that ¼ is the same as
direct the learning 251. Which of the 0.25. They use this
experience becomes following are effective relationship to determine
b. The farther you are methods in teaching that 0.15 and 0.20 are
from the bottom, the student critical reading slightly less than ¼.
more direct the learning skills? Which of the following
experience becomes I. Interpret editorials concept/s is/are being
c. The close you are to II. Read and interpret taught?
the base, the more three different movie a. Numeration skills
indirect the learning reviews b. Place value of
experience become III. Read a position paper decimals
d. The closer you are to and deduce underlying c. Numeration skills for
the base, the more direct assumptions of the decimal and relationship
the learning experience position papers between fractions and
becomes ***** a. II and III c. I and II decimals *****
250. To teach the b. I and III d. I, II and d. Relationship between
democratic process to the III***** fractions and decimals
pupils. Santo Domingo 252. Here is a test item: 254. To nurture student’s
Elementary School “The improvement of creativity, which activity
decided that the election basic education should should the teacher avoid?
of class officers shall be be the top priority of the a. Ask “what if” questions
patterned after local Philippine government. b. Ask divergent thinking
elections. There are Defend and refute the questions
qualities set for position.” c. Emphasize the need to
candidates, limited period Under what type of give right answer *****
for campaign, rules for question does this test d. Be open to “out-of-this
posting campaign item fall? world” ideas
a sense of continuity ***** can read
255. After reading an b. Introduce a new lesson d. Children who have
essay, Teacher Bebe c. Reflect on how she gained mastery over
wants to help sharpen presented the previous basic skills are more
her students’ ability to lesson motivate to read*****
interpret. Which of these d. Determine who among
activities will be most her pupils are studying 259. Bruner’s theory on
appropriate? intellectual development
a. Drawing conclusions 258. Research says that moves from enactive to
b. Making inferences ***** mastery experiences iconic and symbolic
c. Getting the main idea increase confidence and stages. Applying Bruner’s
d. Listing facts separately willingness to try similar theory, how would you
from opinion or more challenging tasks teach?
such as reading. What a. Begin with the abstract
256. What is the best way does this imply for b. Be interactive in
to develop math children reading approach
concepts? performance? c. Begin with the concrete
a. Solving problems using a. Children who have *****
multiple approaches mastered basic skills are d. Do direct instruction
b. Solving problems by more likely to be less
looking for correct answer motivated to read 260. Teacher Nene
c. Learning math as because they get fed up asked this question:
applied to situations such with too much reading “What conclusion can you
as being a tool of science b. Children who have not draw based on your
***** mastered the basic skills observation?” Nobody
d. Solving problems by are more likely to be raised a hand so she
applying learned formulas motivated to read in order asked another question:
257. Teacher Cita, an to gain mastery over “Based on what you
experienced teacher, basic skills observe, what can you
does daily review of past c. Children who have a now say about the
lessons in order to high sense of self- reaction of plants to light.”
________. confidence are not What did Teacher Nene
a. Provide her pupils with necessarily those who do?
a. Redirecting c. a. Role play ***** c. students the important of
Repeating Exhibit good grades
b. Probing d. b. Field trip d. Game b. By making my students
Rephrasing ***** 263. The mode of a score feel my authority over
distribution is 25. This them
261. Under no means that: c. By making them feel
circumstance shall a a. There is no score of 25 a sense of belongingness
teacher be prejudiced nor b. Twenty five (25) is the and acceptance
discriminatory against score that occurs most d. By making my students
any learner according to ***** feel I know what I am
the Code of Ethics. When c. Twenty five is the talking about *****
is a teacher prejudice average of the score 266. Which may help an
against any learner? distribution adolescent discover his
a. When he makes a d. Twenty five is the identity?
nearsighted pupil sit at score that occurs least a. Parents pushing in to
the front 264. The following follow a specific path
b. When he considers characterize a child- b. Relating to people
multiple intelligences in centered kindergarten c. Decision to follow one
the choice of his teaching except: path only
strategies a. Focus on the d. Exploring many
c. When he makes a education of the whole different roles in a healthy
farsighted pupil sit at the child manner *****
back b. Importance of play in 267. Which terms refers
d. When he refuses a development to a teacher helping a
pupil with a slight c. Extreme orientation on colleague grow
physical disability in academic***** professionally?
class***** d. Emphasis on individual a. Technology transfer
uniqueness b. Peer mentoring *****
262. Which learning 265. As a classroom c. Facilitating
activity is most manager, how can you d. Independent study
appropriate if teacher’s exhibit expert power on 268. What primary
focus in attitudinal the first day of school? criterion should guide a
change? a. By citing to my teacher in the choice of
instructional devices? d. Asking divergent explore their feelings and
a. Novelty c. question ***** ideas freely*****
Appropriateness***** 272. For grades to be b. Come up with highly
b. Cost d. (made) valid indicators of competitive games where
Attractiveness students’ achievements, winners will feel happily
269. Some of your which process should be c. Establish this
students don’t seem to observed? discipline of being
like you as their teacher. a. Adopting letter grades judgmental in attitude
If you will regard the such as A, B, C, D d. Tell the students to
situation, on the level of b. Explaining the participate in class
the ego, what will you meaning of grades activities or else they
most likely think about? c. Defining the course won’t receive plus points
a. Why should I care if objectives as intended in class recitation
they like me or not learning outcomes ***** 275. Research on
b. To hell with them d. Giving objective type of Piagetian tasks indicates
c. What’s wrong have I test that thinking becomes
done to deserve this? 273. The claim of a more logical and abstract
d. What is it about me benefactor to the as children reach the
that they do not like? ***** gratitude of his protégé is formal operations stage.
270. Which of the an example of a (an): What is an educational
following is considered a a. Acquired right c. implication of this finding?
peripheral device? Imperfect right ***** a. Expect hypothetical
a. Printer***** c. CPU b. Perfect right d. reasoning for learners
b. Keyboard d. Monitor Alienable right between 12 to 15 years
271. Which questioning 274. Which appropriate of age *****
practice will promote teaching practice flows b. Learners who are not
more class interaction? from this research finding capable of logical
a. Asking rhetorical on the brain: “The brain’s reasoning from ages 8 to
question emotional center is tied 11 behind in their
b. Rejecting wrong its ability to learn.” cognitive development
answer a. Create a learning c. Engage children in
c. Focusing on environment that analogical reasoning as
convergent question encourages students to early as preschool to train
them for higher order of questions except: d. “Porma” over
thinking skills (HOTS) a. To stimulate learners substance*****
d. Let children be children to ask questions 282. Which one should a
276. A mathematics test b. To call the attention of teacher avoid to produce
was given to all Grade V an inattentive student an environment
pupils to determine the ***** conducive for learning?
contestants for the math c. To arouse interest a. Games c. Tests
quiz bee. Which and curiosity b. Seat plan d. Individual
statistical measure d. To teach via student competition *****
should be used to identify answers 283. Between pursuing a
the top 15? 280. What is not a sound college course where
a. Percentage score ***** purpose in making there is no demand and a
b. Mean percentage questions? vocational course which
score a. To remind students of is highly in demand, the
c. Quartile score a procedure Filipino usually opts for
d. Percentile score b. To probe deeper after the college course. Which
277. Which is the true an answer is given Filipino trait is shown?
foundation of the social c. To encourage self- a. Interest to obtain a skill
order? reflection b. Penchant for a college
a. Strong, political d. To discipline a bully in diploma *****
leadership class***** c. Desire for
b. The reciprocation of 281. A student passes a entrepreneurship
rights and duties ***** book report written but d. Appreciation of manual
c. Equitable distribution ornately presented in a labor
of wealth folder to make up for the 284. Which of the
d. Obedient citizenry poor quality of the book following does extreme
278. With which is true content. Which Filipino authoritarianism in the
authority equated? trait does this practice home reinforce in pupils?
a. Service***** c. prove? a. Sense of initiative
Suppression a. Art over science b. Dependence on other
b. Power d. Coercion b. Art over academic for direction *****
279. The following are c. Substance over c. Ability for self-
sound specific purposes “porma” direction
d. Creativity in work plays c. Mobile teacher
285. Teacher Bart wants b. Associative and d. Sine’s skwela
his students to master the cooperative plays ***** 290. Referring to the
concept of social justice. c. Associative and characteristics of the
Which series of activities onlookers plays latest Basic Educational
will be most effective? d. Cooperative and Curriculum which does
a. Pre-teaching > posttest solitary plays not belong to the group?
> re-teaching of 288. How can you exhibit a. More flexible
unlearned concept > referent power on the first b. Less prescriptive
posttest day of school? c. More
b. Pre-test > teaching > a. By making the compartmentalized *****
posttest students feel you know d. More integrated
c. Review > pretest > what you are talking 291. If student’s
teaching > posttest***** about inappropriate behavior is
d. Teaching > posttest b. By telling them the low level or mild and that
286. To provide for importance of good it appears that the
individual differences how grades misbehavior will not
is curriculum designed? c. By reminding your spread to others, it is
a. Minimum learning students your authority sometimes best for the
competencies are over them again and teacher not to take notice
included again of it. What influence
b. Realistic and d. By giving your students technique is this?
meaningful experiences a sense of belonging and a. Planned ignoring *****
are provided ***** acceptance ***** b. Antiseptic bouncing
c. Some degree of 289. To ensure that all c. Proximity control
flexibility is provided Filipino children are d. Signal interference
d. Social skills are functionally literate, which 292. Which among the
emphasized mechanism is meant to following objectives in the
287. Which types of play reach out to children who psychomotor domain is
is most characteristic of a are far from a school? highest in level?
four- to six-year-old a. A school in every a. To distinguish distant
child? barangay ***** and close sounds
a. Solitary and onlooker b. Multi-grade classroom b. To contract a muscle
c. To run a 100-meter approachable.” Which learn
dash power does Teacher b. He like to learn d.
d. To dance the basic Monica possess? Needs to learn
steps of the waltz***** a. Legitimate power 298. When a significantly
293. Which material b. Expert power greater number from the
consists of instructional c. Referent power***** lower group gets a test
units that cater to varying d. Reward power item correctly, this implies
mental level pupils? 296. Kounin claims “with- that the test item:
a. Plantilia it-ness” is one of the a. Is not highly reliable
b. Multi-level materials characteristics of an *****
***** effective classroom b. Is not very valid
c. Multi-grade materials manager. Which among c. Is highly reliable
d. Minimum learning the following is a sign of d. Is very valid
competencies “with-it-ness”? 299. Which statement
294. Which statement on a. Giving attention to about guidance is false?
counseling is false? students having difficulty a. The classroom teacher
a. For counseling to be with school work is not part of the school
successful, the counselee b. Seeing only a portion guidance program since
is willing to participate in of the class but she is not trained to be a
the process intensively guidance counselor *****
b. The ultimate goal of c. Knowing where b. Guidance embraces
counseling is greater instructional materials are curriculum, teaching,
happiness on the part of kept supervision and all other
the counselee***** d. Aware of what’s activities in school
c. Counseling is the happening in all part of c. Guidance is a
program that includes the classroom***** function of the entire
guidance 297. In the K-W-L school
d. The school counselor technique, K stands for d. A guidance program is
is primarily responsible of what the pupils already inherent in every school
counseling knows, W for what he 300. A teacher should not
295. Arianna describes wants to know and L for be a slave of his lesson
Teacher Monica as “fair, what he: plan. This means that:
caring and a. Learned c. Failed to a. A teacher must be
willing to depart from her surely will not like availability of funds
lesson plan if students someone to give you a d. Required for renewal
are interested in death threat in order to of professional license
something other than her pass 304. To ensure high
intended lesson ***** 302. Teacher Alessandra standards of teachers’
b. A lesson plan must be knows of the illegal personal and professional
followed by a teacher no activities of a neighbor development, which of
matter what but keeps quiet in order the following measures
c. A teacher must be not to be involved in any must be implemented?
ready to depart from her investigation. Which I. A school head plans
lesson plan if she foundational principle of the professional
remembers something morality does Teacher development of his/her
more interesting than Alessandra fail to apply? teachers.
what she earlier planned a. Always do what is II. Every teacher
d. Teacher is the best right***** formulates his/her own
lesson plan designer b. The end does not professional development
301. With a death threat justify the means plan.
over her head, Teacher c. The end justifies the III. The implementation of
Donita is directed to pass means what is learned in training
an undeserving student. d. Between two evils, do must be monitored.
What will a utilitarianist the lesser evil a. I only c. II and III*****
do? 303. You are very much b. I and III d. II only
a. Pass the student, why interested in a quality 305. As a community
suffer the threat? professional development leader, which of the
b. Pass the student. That program for teachers. following should a
will be of use to the What characteristic teacher not do?
student, his parents and should you look for? a. Solicit donation from
you***** a. Prescribe by top philanthropists in the
c. Don’t pass him; live by educational teachers community
your principle of justice. b. Responsive to b. Support effort of the
You will get reward, if not identified teacher’s community to improve
in this life, in the next needs***** their status in life
d. Don’t pass him. You c. Dependent on the c. Make herself aloof to
ensure that her decisions professional education 310. The following are
will not be influenced by characteristics of
the community 308. Large class size in interdisciplinary teaching
politics***** congested cities is a except:
d. Play an active part in common problem in our a. Allows learners to see
the activities of the public schools. Which connectedness between
community measure/s have schools things
306. In a highly pluralistic taken to offset the effects b. Provides learning
society, what type of of large class? opportunities in a real-
learning environment is I. The deployment of world context
the responsibility of the more teachers c. Eliminates boundaries
teacher? II. The implementation of between content area
I. Safe III. Secure 1:1 pupil textbook ratio d. Discussion from a
II. Gender-biased III. The conduct of single perspective*****
a. I and II c. II only morning and afternoon 311. For more efficient
b. I, II and III d. I and sessions and effective
III***** a. I, II and III***** c. III management of school as
307. A teacher is said to only agents of change, one
be “trustee of the cultural b. I and II d. II only proposal is for the DepEd
and educational heritage 309. The failure of to cluster remote stand-
of the nation and is under independent study with alone schools under one
obligation to transmit to most Filipino students lead school head. Which
learners such heritage”. may be attributed to factor has the strongest
Which practice makes the students’? influence on this
teacher fulfill such a. Unpreparedness for proposal?
obligation? schooling***** a. Psychological c.
a. Use interactive b. Ambivalence Geographical*****
teaching strategies c. High degree of b. Historical d. Social
b. As a class, study the independence 312. What does the
life of Filipino heroes***** d. High degree of acronym EFA imply for
c. Use the latest independence on schools?
educational technology authority a. The acceptance of
d. Observe continuing exclusive schools for
boys and for girls development points to the requires of every
b. The stress on the need to do more in professional teacher for
superiority of formal school. This is base on him/her to be competent
education over that of which pillar of learning? is in line with which pillar
alternative learning a. Learning to do of learning?
system b. Learning to know a. Learning to know*****
c. Practice of inclusive c. Learning to live b. Learning to be
education***** together c. Learning to live
d. The concentration on d. Learning to be***** together
formal education system d. Learning to do
315. The schooling 317. Widespread abuse
313. The wide incidents in school of Mother Earth prompted
acceptance of “bottom campuses abroad have schools to teach
up” management style made school to rethink sustainable development.
has influenced schools to the curriculum. Which is Which one does this
practice which believed to counteract prove about schools?
management practice? such incidents and so is a. The curricula of
a. Exclusion of politicians being introduced in schools are centered on
from the pool of guest schools? Mother Earth
speakers during I. Inclusion of socio- b. Environment factors
graduation exercises emotional teaching influence the school as
b. Prescription of what II. The emphasis on the an agent of change*****
ought to be done from the concept of competition c. Schools can easily
Center Office against self and not integrate sustained
c. Involvement of against others development in their
students, parents, III. Focus on academic curriculum
teachers, and community achievement and d. Sustained
in school planning***** productivity development cannot be
d. Allowing schools to do a. I and III c. I and II effectively taught in the
what they think is best b. II and III d. I, II and classroom
314. Material III***** 318. Which pillar of
development at the learning is aimed at the
expense of human 316. The specialization holistic development of
man and his complete d. No, it is simply an act b. Yes, it somewhat
fulfillment? of benevolence from eliminates gender
a. Learning to live President GMA disparities
together 321. As provided for the c. Yes, it supports
b. Learning to be***** Educational Act of 1982, equitable access to basic
c. Learning to know how are the institutions of education*****
d. Learning to do learning encouraged d. No, it does not support
319. A father tells his setting higher standards parent of adult education
daughter “You are a of equality over the
woman. You are meant minimum standards 323. “Specialization is
for the home and so for required for state knowing more and more
you, going to school is recognition? about less and less.”
not necessary.” Is the a. Voluntary accreditation Hence, it is better to be a
father correct? b. Continuing generalist, claims
a. It depends on the Professional Teacher Fonda. Which
place where the daughter Education***** Philosophy does Teacher
and the father live c. Granting of Special Fonda subscribe to?
b. No, there is gender Permit a. Existentialism c.
equality in education***** d. Academic freedom Essentialism*****
c. Yes, women are meant b. Perennialism d.
to be a mother only 322. Despite of Progressivism
d. No, today women can opposition from some
take on the jobs of men school official, DepEd 324. Mencius believed
has continuously that all people are born
320. Is there a legal basis enforced the “no good. This thought on the
for increasing the collection of fees” policy innate goodness of
teacher’s starting salary during enrolment period people makes it easier to
to PHP18,000 a months? in public schools. In this ________ our pupils.
a. No, it is a gift to policy in accordance with a. teach c. like
teachers from Congress EFA goals? b. respect *****
b. Yes, R.A 7836 a. No, it violates the d. motivate
c. Yes, the Philippine mandate of equality 325. A student complains
Constitution***** education to you about his failing
grade. When you III. Teacher Catherine yes after promotion
recomputed you found secretly gives way to a b. No, it is simply earning
out that you committed special favor (e.g. add 2 MA units for
an error in his grade points to grade) promotion*****
computation. Your requested by student c. It depends on the
decision is not to accept Alex who is vying for school she is enrolled in
the erroneous honors. d. Yes, just enrolling in an
computation before the a. II and III c. I and II MA program is already
student and so leave the b. I, II and III***** d. I and professional growth
failing grade as is for fear III 329. If Teacher Analiza
that you may lose 327. Which is/are in asks more higher-order
credibility. Is this morally accordance with the questions, she has to ask
right? principle of pedagogical more ________
a. No, the reason for not competence? questions.
accepting the error before I. Communication of a. fact c. convergent
the students is flimsy objectives of the course b. close d. concept*****
b. No, the end does not to student. 330. Misdemeanor has a
justify the means***** II. Awareness of “ripple effect.” This
c. Yes, the end justifies alternative instruction implies that as a
the means strategies. classroom manager, a
d. Yes, as a teacher you III. Selection of teacher:
must maintain your appropriate methods of a. reinforces positive
credibility instruction. behavior
326. Which violate(s) the a. I and III c. III only b. responds to
principle of respect? b. I, II and III***** d. II misbehavior promptly*****
I. Teacher Ana tells her and III c. is consistent in her
students that what 328. To earn units for classroom management
Teacher Beth taught is promotion, Teacher Flora practice
wrong. pays her fee but does not d. count 1 to 10 before
II. To retaliate, Teacher attend class at all. Does she deals with a
Beth advises students not this constitute misbehaving student
to enroll in Teacher Ana’s professional growth? 331. Based on Edgar
class. a. Not immediately but Dale’s “Cone of
Experience,” which no longer on task strives to draw
activity is farthest from c. Make eye contact to participation of every
the real thing? communicate what you student into her
a. Watching demo c. wish to communicate classroom discussion.
Video disc d. Move closer to the Which of these student
b. Attending exhibit d. child to make him feel needs is she trying to
Viewing images***** your presence address? The need to
332. The students of 334. When Teacher Pearl _______.
Teacher Yue scan an tries to elicit clarification a. feel significant and be
electronic encyclopedia, on a student response or part of a group
view a film on subject, or solicits additional b. show one’s oral
look at related topics at information, which of abilities to the rest of the
the touch of a button right these should be use? class
there in the classroom. a. Directing c. c. get everything and be
Which device/s does Structuring part of a group
teacher Yue’s class b. Probing d. Cross d. be creative
have? examining 338. To be an effective
a. Teaching 335. Which priority classroom manager, a
machines***** criterion should guide a teacher must be friendly
b. CD teacher in the choice of but must at the same
c. Video disc instructional devices? time be _________.
d. Videotaped lesson a. Novelty c. a. confident c. analytical
333. Which is an Attractiveness b. business-like***** d.
inappropriate way to b. Cost d. buddy-buddy
manage off-task Appropriateness
behavior? 336. Which learning 339. Which software is
a. Redirect a child’s activity is most needed when one wants
attention to task and appropriate if a teacher’s to perform automatic
check his progress to focus is attitudinal calculations on numerical
make sure he is change? data?
continuing work a. Fieldtrip c. Role play a. Database
b. Stop your class activity b. Exhibit d. Game b. Spreadsheet
to correct a child who is 337. Teacher Hannah Program*****
c. Microsoft Word and acceptance*****
d. Microsoft Powerpoint 342. Which of these is c. making them feel you
one of the ways by which know what you are
340. Which of the the internet enables talking about
following questions must people to browse d. reminding your
be considered in documents connected by students your authority
evaluating teacher-made hypertext links? over them again and
materials? a. URL***** c. Welcome again
a. In the material new? page 345. I would like to use a
b. Does the material b. Browser d. World Wide model to emphasize
simulate individualism? Web particular part. Which of
***** 343. Which these would be most
c. Is the material characteristics must be appropriate?
expensive? primarily considered as a a. Regalia c. Stimulation
d. Is the material cheap? choice of instructional b. Audio recording d.
341. Kounin claims that aides? Mock up*****
“with-it-ness” is one of the a. Stimulate and maintain
characteristics of an students interests 346. What must Teacher
effective classroom b. Suited to the lesson Luke do to ensure orderly
manager. What is one objectives***** transitions between
sign of “with-it-ness”? c. Updated and relevant activities?
a. Giving attention to to Filipino setting a. Have the materials
students who are having d. New and skillfully ready at the start of the
difficulty with school work made activity*****
b. Aware of what’s 344. You can exhibit b. Allow time for the
happening in all parts of referent power on the first students to socialize in
the classroom***** day of school by between activities
c. Seeing only a portion __________. c. Assign fewer exercise
of the class but a. telling them the to fill the allotted time
intensively importance of good d. Wait for students who
d. Knowing where grades lag behind
instructional materials are b. giving your students a 347. The task of setting
kept sense of belongingness up routine activities for
effective classroom a misbehaving student that provide an overall
management is a task d. She stops misbehaving picture of the lesson *****
that a teacher should without disrupting lesson d. Independent
undertake: flow *****
a. as soon as the 352. One’s approach to
students have adjusted to 350. If curriculum is teaching is influenced by
their schedules designed following the Howard Gardner’s MI
b. on the very first day of traditional approach, Theory. What is he/she
school***** which feature(s) challenged to do?
c. every day at the start apply(ies)? I. To come up with 9
of the session I. The aims of the different ways of
d. every homeroom day curriculum are set by approaching lesson to
professionals and cater to the 9 multiple
348. What principle is experts. intelligence
violated by overusing the II. Interested groups II. To develop all
chalkboard, as though it (teachers, students, student’s skill in all nine
is the only education communities) are intelligences
technology available? assumed to agree with III. To provide worthwhile
a. Isolated use c. the aims of the activities that
Variety***** curriculum. acknowledge individual
b. Flexibility d. Uniformity III. Consensus building in difference in children
not necessary. a. I, II and III c. II only
349. Teacher Sandra a. III only c. I and II***** b. II and III d. III only*****
uses the low-profile b. I, II and III d. I and III
classroom control 353. If my approach to
technique most of the 351. If you make use of my lesson is behaviorist,
time. What does this the indirect instruction what features will
imply? method, you begin your dominate my lesson?
a. She is reactive in her lesson with: I. Copying notes III.
disciplinary orientation a. Guided practice Lecturing
b. She manages pupils b. A review of previous II. Reasoning IV.
personalities day’s work Demonstration
c. She reacts severely to c. Advance organizers a. III, IV c. I, II, III, IV
b. I, III, IV d. II, III, IV***** localization experience becomes
354. You practice c. The trend towards b. The farther you are
inclusive education. participatory curriculum from the bottom, the
Which of these applies to development more direct the learning
you? d. The shift in the experience becomes
I. You accept every paradigm of curriculum c. The closer you are
student as full and valued development from a from the base, the more
member of the class and process-oriented to a indirect the learning
school community product-oriented one***** experience becomes
II. Your special attention 356. You choose d. The closer you are
is on learners with cooperative learning as a from the base, the more
specific learning or social teaching approach. What direct the learning
needs thought is impressed on experience becomes*****
III. Your address the your students? 358. “When more senses
needs of the class as a a. Interaction is a must, are stimulated, teaching
whole within the context but not necessarily face and learning become
of the learners with to-face interaction more effective.” What is
specific learning or social b. Student’s success an application of this
needs depends on the success principle?
a. II only c. I only of the group***** a. Appeal to student’s
b. I and II d. I and III***** c. Student’s individuality sense of imagination
355. School curriculum evaluates how effectively b. Use multisensory
reflects the world’s their group worked aids*****
economic and political d. The accountability for c. Make your students
integration and learning is on the group touch the instructional
industrialization. What not on the individual material
does these points in 357. Which statement d. Use audiovisual aids
curriculum development? applies correctly to Edgar because the eyes and the
a. The trend towards the Dale’s “Cone of ears are the most
classical approach to Experience”? important senses in
curriculum development a. The farther you are learning
b. The trend towards the from the base, the more 359. Which is a
globalization and direct the learning classroom application of
the theory of “operant supported by the data”. make her teaching easier
conditioning”? Under what type of by having less talk
a. Help student see the question does this item b. The teacher
connectedness of facts, fall? emphasizing reading and
concepts, and principles a. Convergent c. writing skills
b. Create a classroom Application c. The teacher is applying
atmosphere that elicits b. Evaluative***** d. Bloom’s hierarchy of
relaxation Divergent cognitive learning
c. Reinforce a good d. The teacher is
behavior to increase the 362. I want to teach teaching in a variety of
likelihood that the learner concepts, patterns and ways because not all
will repeat the abstractions. Which students learn in the
response***** method will be most same manner*****
d. Make students learn by appropriate?
operating manipulatively a. Discovery c. Direct 364. Teacher Georgina,
360. Read the following instruction***** an experienced teacher,
teacher-student situation. b. Indirect instruction does daily review of past
TEACHER: Why is the d. Problem solving lessons in order to:
process called a. introduce a new lesson
photosynthesis? 363. Teacher Atilla b. reflect on how she
STUDENT: I don’t know. teaches English as a presented the previous
Which questioning Second Language. She lessons
technique should be the uses vocabulary cards, c. provide her pupils with
teacher be using? fill-in-the-blanks a sense of continuity*****
a. Clarification c. sentences, dialogues, d. determine who among
Prompting***** dictation and writing her pupils are studying
b. Multiple response d. exercises in teaching a
Concept review lesson about grocery 365. I combined several
shopping. Based on this subject areas in order to
361. Here is the test item. information, which of the focus on a single concept
“From the data presented following is a valid for interdisciplinary
in the table, form conclusion? teaching. Which strategy
generalizations that are a. The teacher wants to did I use?
a. Reading-writing activity three different movie that ¼ is the same as
b. Thematic reviews 0.25. He use this
introduction***** III. Read a position paper relationship to determine
c. Unit method and deduce underlying that 0.15 and 0.20 are
d. Problem-centered assumptions of the slightly less than ¼.
learning position papers Which of the following
a. II and III c. I and II concept/s is/are being
366. To teach the b. I and III d. I, II and taught?
democratic process to the III***** a. Numeration skills of
pupils, Batongmalaki 368. Here is a test item: decimals and
Elementary School “The improvement of relationships between
decided that the election basic education should fractions and
of class officers shall be be the top priority of the decimals*****
patterned after local Philippine government. b. Numeration skills
elections. There are Defend or refute this c. Place value of
qualifications set for position.” decimals
candidates, limited period Under what type of d. Relationship between
for campaign and rules question does this test fraction and decimals
for posting campaign item fall? 371. What is the best way
materials, etc. Which of a. Low-level c. Analysis to develop math concept?
the following did the b. Evaluative***** d. a. Solving problems using
school use? Convergent multiple approaches
a. Symposium c. Pole 369. When I teach, I often b. Solving problems by
playing engage in brainstorming. looking for correct answer
b. Simulation***** d. Which do I avoid? c. Learning math as
Philips 66 a. Break down barriers applied to situations,
367. Which among the b. Selectively involves such as being a tool of
following are effective pupils***** science*****
methods in teaching c. Increase creativity d. Solving problems by
student critical reading d. Generate many ideas applying learned formulas
skills? 370. Teacher Sammy 372. After the reading of
I. Interpret editorial teaches a lesson in which a selection in the class,
II. Read and interpret students must recognize which of these activities
can enhance students’ world” ideas b. II d. III
creativity? 375. Teacher Rodel 378. When small children
I. Reader’s theater wants to develop his call animals “dog”, what
II. Reading aloud students’ creativity. process is illustrated on
III. Silent reading Which type of questions Piaget’s cognitive
a. I and II c. I will be most appropriate? development theory?
only***** a. Synthesis questions a. Reversion c.
b. II only d. III only b. Fact questions Accommodation
373. Teacher Carlo, a c. “What if…” b. Assimilation***** d.
Reading teacher, advised questions***** Conservation
the class to “read d. Analysis questions 379. Researchers found
between the lines”. What 376. Floramay enjoyed that when a child is
does she want his pupils the roller coaster when engaged in a learning
to do? she and her family went experience a number of
a. Determine what is to Disneyland. The mere areas of the brain are
meant by what is sight of a roller coaster simultaneously activated.
stated***** gets her excited. Which Which of the following
b. Make an educated theory explains is/are implication/s of this
guess Floramay’s behavior? research finding?
c. Apply the information a. Operant conditioning I. Make use of field trips
being read b. Pavlovian and guest speakers
d. Describe the conditioning***** II. Do multicultural units
characters in the story c. Social learning theory of study
374. To nurture students’ d. Attribution theory III. Stick to the “left brain
creativity, which activity 377. According to Freud, and right brain” approach
should a teacher avoid? with which should one be a. I and III c. I and II*****
a. Ask “What if…” concerned if he/she has b. I only d. II only
questions to develop in the students 380. My problem is there
b. Emphasize the need a correct sense of right are too many topics to
to give right answers***** and wrong? cover and I may not able
c. Ask divergent thinking I. Super-ego II. Ego to finish before classes
questions III. Id end in March. Which
d. Be open to “out-of-this- a. I and II c. I***** approach when used can
help solve my problem? relevant to students’ activities will be most
a. Thematic***** c. basic human needs***** appropriate?
Experimental approach b. Make teaching-leaning a. Drawing conclusions
b. Constructive d. interactive b. Making inferences*****
Direct instruction c. Avoid giving c. Getting the main idea
381. You want your assignments d. Listing facts separately
students to answer the d. Organize a curriculum from opinion
questions at the end of a in a spiral manner 386. Read the following
reading lesson. “What did 383. Nadia exhibit fear then answer the question.
I learn?”, “What still response to freely A man and his son are
puzzles me?”, “What did I roaming dogs but does driving in a car. The car
enjoy, hate and not show fear when a dog crashes into a tree, killing
accomplish in the class is on a leash or confined the father and seriously
today?” and “How did I to a pen. Which injury his son. At the
learn from the conditioning process is hospital, the boy needs to
lesson?”.Which of the illustrated? have surgery. Looking at
following are you asking a. Extinction c. the boy, the doctor says
them to do? Acquisition (telling the truth), “I
a. Work on an b. Generalization d. cannot operate on him.
assignment Discrimination***** He is my son. How can
b. Make journal entry***** 384. Based on Freud’s this be?
c. Work on a drill theory, which operate/s ANSWER: The doctor is
d. Apply what they when a student strikes a the boy’s mother.
learned classmate at the height of The above brain twister
382. William Glasser’s anger? helps develop critical
control theory states that a. Ego c. Id and Ego reading skills. Which
behavior is inspired by interact activity was used?
what satisfies a person b. Id***** d. Superego a. Comparing c.
want at any given time. 385. After reading an Inferring meaning*****
What then must a teacher essay. Teacher Beatrice b. Classifying d. Looking
do to motivate students to wants to help sharpen for cause and effect
learn? her students’ ability to 387. Research says that
a. Make schoolwork interpret. Which of these mastery experiences
increase confidence and I. Sharing the excitement d. Classical
willingness to try similar of read-aloud conditioning*****
or more challenging tasks II. Showing their passion 391. Which is/are the
as reading. What does for reading basic assumption/s of
this imply for children’s III. Being rewarded to behaviorists?
reading performance? demonstrate the value of I. The mind of newborn
a. Children who have not reading child is a blank state
mastered the basic skills a. II and III c. I, II and II. All behaviors are
are more likely to be III***** determined by
motivated to read in order b. I and II d. II only environmental events
to gain mastery over 389. Bruner’s theory on III. The child has a certain
basic skills intellectual development degree of freedom not to
b. Children who have moves from enactive to allow himself to be
mastered basic skills are iconic and symbolic shaped by his
more likely to be less stages. Applying Bruner’s environment
motivated to read theory. How would you a. III only c. II only
because they get fed up teach? b. I and II d. I and III*****
with too much reading a. Be interactive in 392. If a student is
c. Children who have approach encourage to develop
gained mastery over b. Begin with the abstract himself to the fullest and
basic skills are more c. Begin with the must satisfy his hierarchy
motivated to read***** concrete***** of needs, the highest
d. Children who have a d. Do direct instruction needs to satisfy
high sense of self- 390. A person who has according to Maslow is
confidence are not painful experiences at the ________.
necessarily those who dentist’s office may a. Psychological need c.
can read become fearful at the Belongingness
388. The value that mere sight of the dentist’s b. Self-actualization*****
students put on reading is office building. Which d. Safety needs
critical to their success. In theory can explain this? 393. In a Social Studies
what way/s can teachers a. Generalization class. Teacher Ina
inculcate his value for b. Operant Conditioning presents a morally
reading? c. Attribution theory ambiguous situation and
asks student what they IV through the correct
would do. On whose 396. According to movements. As his
theory is Teacher Ina’s Erikson, what years are students become more
technique based? critical for the skillful, he stands back
a. Bandura c. development of self- from the man and gives
Kohlberg***** confidence? verbal feedback about
b. Piaget d. Bruner a. High school years how to improve. With
394. Teacher Violy is b. Elementary school Vygotsky’s theory in
convinced that whenever years***** mind, what did Teacher
a student performs a c. College years Henry do?
desired behavior, provide d. Preschool years a. Apprenticeship
reinforcement and soon 397. Which of the c. Peer interaction
the student learns to following does not b. Guided participation
perform the behavior on describe the development d. Scaffolding*****
his own. On which of children aged 11 to 399. What does Gagne’s
principle is Teacher 13? hierarchy theory propose
Violy’s conviction based? a. They exhibit increased for effective instruction?
a. Environmentalism objectivity in thinking a. Be concerned with the
c. Cognitivism b. Sex difference in IQ socio-emotional climate
b. Behaviorism***** d. become more in the classroom
Constructivism evident***** b. Sequence
395. In Bandura’s social c. They shift from instruction*****
learning theory, it states impulsivity to adaptive c. Teach beginning with
that children often imitate ability the concrete
those who: d. They show abstract d. Reward good behavior
I. have substantial thinking and judgment 400. Which appropriate
influence over their lives 398. Teacher Henry teaching practice flows
II. belong their peer group begins a lesson on from this research finding
III. belong to other race tumbling, demonstrating on the brain: “The brain’s
IV. are successful and front and back emotional center is tied
seem admired somersaults in slow into its ability to learn”.
a. IV only c. I and II motion and physically a. Establish the discipline
b. I and IV ***** d. II and guiding his students of being judgmental in
attitude c. Let children be characterize/s a learning
b. Come up with highly children environment that
competitive games where d. Expect hypothetical promotes fairness among
winners will feel happy reasoning for learners learners of various
c. Create a learning between 12 to 15 years cultures, family
environment that of age***** background and gender?
encourages students to I. Inclusive III. Gender-
explore their feeling and 402. Research says: sensitive
ideas freely***** “People tend to attribute II. Exclusive
d. Tell the students to their successes to a. I only c. I and III
participate in class internal causes and their b. III only d. II and III
activities or else won’t failures to external
receive plus points in causes.” Based on this 404. Which of the
class recitation finding, what should be following steps should be
taught to students for completed first in
401. Research on them to be genuinely planning an achievement
Piagetian tasks indicates motivated to succeed? test?
that thinking becomes a. Tell them the research a. Define the instructional
more logical and abstract finding when applied will objective*****
as children reach the make them genuinely b. Set up a table of
formal operation stage. motivated specialization
What is an educational b. Convince them that c. Select the types of
implication of this finding? genuine motivation is the test items to use
a. Engage children in only factor that matters d. Decide on the length of
analogical reasoning as for a person to succeed the test
early as preschool to train c. Make them realize that 405. The computed r for
them for higher order failure is a part of life***** scores in Math and
thinking skills (HOTS) d. Make them realize that Science is 0.92. What
b. Learners who are not both success and failure does this mean?
capable of logical are more a function of a. Math score is positive
reasoning from ages 8 to internal causes related to Science
11 lag behind in their score*****
cognitive development 403. Which b. The higher the Math
score, the lower the skills. Which has the a. Below average
Science score highest diagnostic value? b. Average*****
c. Math score is not in a. Multiple choice test c. Needs Improvement
any way related to b. Performance test d. Above average
Science score c. Essay test ***** 411. Here is a test item:
d. Science score is d. Completion test
slightly related to Math
score 409. Here is a test item:

406. Which types of test


is most appropriate if
Teacher Yanny wants to
measure student’s ability Why is this test item
to organize thoughts and poor?
ideas? I. The test item does not What makes the multiple
a. Short answer type of pose a problem to the choice type of test poor?
test examinee a. The options are not
b. Extended response II. There are variety of grammatically connected
essay***** possible correct answer to the stem
c. Modified alternative to this item b. The stem fails to
response III. The language used in present a problem*****
d. Limited response the question is not c. There are
essay precise grammatical clues
IV. The blank is near the d. The options are not
407. If I want to hone my beginning of a sentence parallel
student’s meta-cognitive a. I and III c. I and IV 412. If Teacher Betty
ability, which is most fit? b. II and IV ***** d. I and wants to measure her
a. Drill c. Brainstorming II students’ ability to
b. Debate d. Journaling discriminate, which of
***** 410. Joanne’s score is these is an appropriate
within x±1 SD. To which type of test item as
408. I want to test of the following groups implied by the direction?
student’s synthesizing does she belong? a. “Outline the Chapter
on The Cell.” D .40 .41
b. “Summarize the lesson E .63 .07
yesterday.” a. A only c. A and D
c. “Group the following b. B only ***** d. B and E
items according to
shape.” ***** 418. A mathematician’s
d. “State a set of principle test was given to all
that can explain the What is WRONG with this Grade V pupils to
following events.” item? determine the
a. Item is overly specific contestants for the Math
413. A test item has a b. Content is trivial***** Quiz Bee. Which
difficult index of 0.89 and c. Test item is option- statistical measure
a discrimination index of based should be used to identify
0.44. What should the d. There is a cue to the the top 15?
teacher do? right answer a. Mean percentage
a. Reject the item score
b. Revise the item***** 416. A student’s score b. Quartile Deviation
c. Make it a bonus item were as follows: 82, 83, c. Percentile Rank
d. Make it a bonus item 84, 86, 88, 84, 83, 85. d. Percentage Score*****
and reject it The score 84 is the: 419. Use the inbox below
a. Mode c. Median***** to answer the question
414. Which form of b. Average d. Mean that follows:
assessment is consistent Which of the following
with the saying “The 417. Which text form statement is true about
proof of the pudding is in would you choose if you the plot of grades above?
the eating.” want to have a valid and a. The median is a score
a. Contrived c. reliable test based on the of 80 and the range is 60
Traditional table below? b. The median is a score
b. Authentic***** d. Test Form Validity Index of 70 and the range is 60
Indirect Reliability Index c. The median is a score
A .47 .68 of 80 and the range is
415. Here is a test item: B .87 .57 20*****
C .20 .86 d. The median is a score
of 70 and the range is 20 concerned***** (higher order thinking
420. Which can be said c. did not attain his skills)
of Nina who obtained a lesson objective because d. Constructing
score of 75 out of 100 of the pupils’ lack of departmentalized exams
items in a Grammar attention for each subject area.
objective test? d. attained his lesson *****
a. She performed better objective because of his 425. If the scores of your
than 25% of her effective spelling drill test follow a negatively
classmates 422. Which of these can skewed score
b. She answered 75 measure awareness of distribution, what should
items in the test values? you do? Find out
correctly***** a. Sociogram __________?
c. Her rating is 75 b. Moral dilemmas***** a. why your items were
d. She answered 75% of c. Projective techniques easy
the test items correctly d. Rating scales b. why most of the scores
421. The criterion of 423. Marking on a are high*****
success in Teacher normative basis means c. why most of the
Butch’s objective is that that: scores are low
“the pupils must be able a. the normal distribution d. why some pupils
to spell 90% of the words curve should be scored high
correctly”. Student Dave followed***** 426. Principal Gemma is
and 24 others in the class b. some should fall talking about “grading on
spelled only 40 out of 50 c. some get high marks the curve” in a faculty
words correctly while the d. the grading is based meeting. What does this
rest scored 45 and on a present criteria expression refers to?
above. This means that 424. Which process a. A student’s mark
Teacher Butch enhances the compares his
_____________. comparability of grades? achievement to his effort
a. attained his lesson a. Using a table b. A student’s grade or
objective specifications mark depends on how his
b. failed to attain his b. Determining the level achievement compares
lesson objective as far as of difficulty of the tests with the achievement of
the 25 pupils are c. Giving more HOTS other students in a
class***** c. They may not get true come/s from a
c. A student’s grade information because behaviorist?
determines whether or individuals can hide or a. #2 and #4 c. #3 and
not a student attains a disguise feelings***** #4
defined standard of d. They have poor b. #1 and #2 d. #1 and
achievement internal consistency #3*****
d. A student’s mark tells
how closely he is 430. On which
achieving to his potential Situation 1. (#129-131) educational philosophy is
In a faculty meeting, the response #1 anchored?
427. Which tests principle told his teacher: a. Existentialism c.
determine whether “We need to improve our Progressivism
students accept school performance in b. Essentialism d.
responsibility for their the National Achievement Bahaviorism*****
own behavior or pass on Test. What should we
responsibility for their do?” 431. If you leaned toward
own behavior to other The teacher gave varied a progressivist
people? answers as follows: philosophy, with which
a. Locus-of-control 1. Let’s give incentives response would you
tests***** and rewards to students agree?
b. Thematic tests who get a rating of 85%. a. #2 b. #3 c.
c. Stylistic test 2. Let’s teach them to #4***** d. #1
d. Sentence-completion accept complete
tests responsibility for their
performance.
428. Which of the 3. Let’s make the school Situation 2. (#132-134)
following is a weakness environment conducive One principle in the
of self-supporting for learning. utilization of technology
personality checklists? 4. Let’s make use of the of the classroom is
a. Many personality experimental methods of appropriateness of
measures have built-in lie teaching. material or activity.
scales
b. They lack stability 429. Which response/s 432. Teacher Wilson
wants his students to 436. To ask the class any
master the concept of insight derived from the
social justice. Which poem is based on the
series of activities will be theory of:
most effective? a. Realism c.
a. Pretest-teaching- Conditioning
posttest b. Behaviorism d.
b. Pretest-teaching- Constructivism
posttest-re-teaching for Situation 3. (#135-137) 437. On which
unlearned concepts- After reading and assumption about the
posttest***** paraphrasing Robert learner is Mr. Sales’s act
c. Review-pretest- Frost’s “Stopping by the of asking the class to
teaching-posttest Wood on a Snowy share their insight based?
d. Teaching-posttest Evening.” Mr. Sales a. Learners are like
433. Teacher Raymund asked the class to share empty receptacles
likes to show how the any insight derived from waiting to be filled up
launching of spaceships the poem. b. Learners are meant to
takes place. Which of the interact with one another
following materials 435. The class was c. Learners have
available is most fit? asked to share their multiple intelligence and
a. Model b. Mock-up insights about the poem. varied learning styles
*****c. Replica d. Realia The ability to come up d. Learners are
434. Teacher June likes with an insight stems producers of knowledge
to concretize the abstract from the ability to: not only passive
concepts of an atom. He a. analyze the parts of a recipients of information
came up with a concrete whole
presentation of the atom b. evaluate the
by using wire and plastic worthiness of a thing Situation 4. (#138-140)
balls. How would you c. relate and organize Principal Evelyn wants
classify Teacher June’s things and ideas her teachers to apply
visual aids? d. comprehend the constructivism in
a. Chart b. Replica c. subject that is being teaching.
Model ***** d. Realia studied
438. On which triangle in items #1 and
assumption/s is the #2 in column A
principal’s action d. The word “gon” must
anchored? Situation 5. (#141-143) be included in column B
I. Students learn by Study the matching type 442. What is the main
personally constructing of test then answer the 3 defect of this matching
meaning of what is taught questions that follow: test?
II. Students construct and Column A a. The matching type is
reconstruct meaning Column B an imperfect type
based on experiences 1. equilateral triangle b. The items are NOT
III. Students derive A. With 3 equal sides homogeneous*****
meaning from the 2. right triangle B. c. The items quite easy
meaning that the teacher With 5 equal sides d. An obvious pattern is
gives 3. octagon C. Has followed in the answering
a. II only c. I, II and III 90- degree 443. Which should be
b. I and II d. I only 4. pentagon angle done to improve the
5. heptagon D. matching type of test?
439. Which materials will Means many a. Capitalize the items in
her teachers least 6. poly E. with Column A
prefers? 7 sides b. Items in Column A and
a. Controversial issues F. with 8 sides B should be exchanged
b. Open-ended topics c. Drop #6 item in
c. Unquestionable Column A*****
laws***** d. The item in Column A
d. Problem or cases should be increased
440. Which concept/s of
the learner will Principal 441. How can you make
Evelyn not accept? the items homogeneous?
I. “Empty vesse!” a. Increase the number of
II. “Tabula rasa” items in Column B
III. Candle to be lighted b. All items should be on
a. III only c. II only polygons*****
b. I only d. I and II***** c. Remove the word
only Lewin, do you feel the
445. Which statement is pain?
true of the rubric? a. Transition***** c.
a. It is developmental Refreezing
b. It is analytical b. Unfreezing d.
c. It is both holistic and Transition and freezing
developmental***** 449. In the writing of
d. It is holistic performance objectives
446. Which is true of the for mastery learning,
scoring rubric? what is an acceptable
I. It describes criteria of standard of performance
levels of achievement called?
II. It has a rating scheme a. Behavior c. SMART
III. It limit itself to 4 levels b. Condition d. Criterion
of achievement measure*****
a. I and II ***** c. II and 450. Why learning
III activities should be
b. I and III d. I, II and III carefully planned?
444. Which of this/these 447. Which guidelines a. The ability of the
is/are essential in should you follow for teacher to plan is tested
constructing a scoring behavior modification to b. This is expected by
rubric? be effective? pupils
I. Description of criteria a. Subject yourself to c. The accomplishment of
to serve as standard group pressure objectives is dependent
II. Clear descriptions of b. Work on several on the plan *****
performance at each behaviors at the time for d. This is required of a
level a significant change teacher
III. Levels of achievement c. Never use negative
(mastery) reinforcement 451. Which statement on
IV. Rating scheme d. Work on one behavior IQ and EQ is correct?
a. I, II and III c. I, II, III at a time ***** a. EQ has a greater
and IV***** 448. In which phases/s of contribution to
b. I and II d. I change, according to Kurt performance than IQ
b. IQ has a greater is in this subject where c. Correct her and remind
contribution to the learner her torturing one’s own
performance than EQ ____________. pupil for a fee is unethical
c. Blending of both IQ a. Will be taught the *****
and EQ can make a Filipino strengths and d. As a group, report her
difference in performance weaknesses to the principal
***** b. Will demonstrate 455. When Teacher Demi
d. The contribution of IQ practical knowledge and presents a set of data
and EQ to performance is skills gained in the other then asks the students to
dependent on factors like subjects ***** enter a conclusion,
age and gender c. Will be taught the true generalization or a
concept of being pattern of relationship
452. Teacher Nikko “pagkamakabayan” which method does she
helped his students recall d. The biographies of use?
that stalagmites grow on heroes who are a. Process approach
the “ground” while “makabayan” will be b. Type method
stalactites grow on the taught c. Unit method
“ceiling” of a cave by 454. In the faculty room d. Inductive inquiry
associating “G” in everyone is talking about method *****
stalaGmites with ground a teacher who is torturing 456. Which individualized
and “C” in stalaCtites with for a fee from her own teaching method makes
ceiling. What did Teacher pupil who is vying for use of workbooks,
Nikko make use of it? honors. What is the teaching machines or
a. Visual aid professional thing for the computers?
b. Mnemonic device ***** other teachers to do? a. Project method
c. Audio-visual aid a. Talk to the parents of b. Unit method
d. Meaning-maker device the tutee. Tell them what c. Programmed
453. “Makabayan” as a teachers doing is instruction *****
subject in the re- unprofessional d. Inductive inquiry
structured Basic b. Leave her alone, she method
Education Curriculum is might accuse you of 457. I want to elicit more
the “laboratory for life.” meddling in her personal student responses.
What does this mean? It life Which one should I
avoid? to form unions or to strike a. Filipino medical
a. Creating an evaluative can be suppressed in doctors
atmosphere ***** times of national b. Parents
b. Using covert emergency. On what c. The state and of every
responses norm is this based? teacher *****
c. Prompting to covert a. Higher law, inalienable d. School administration
wrong answers to correct rights before alienable 463. In a study
ones b. Wider social order, the conducted, the pupils
d. Giving non-threatening society before the were asked which
comments individual ***** nationality they preferred
458. For practice to be c. Clearer title, the certain if given a choice. Majority
effective, which guideline before the title of the pupils wanted to be
should bear in mind? d. Nobler person, God an American. In this
Practice should _______. before man case, in which obligation
a. Be done in an 461. Which skills should relative to the state are
evaluative atmosphere be taught if Teacher Joey schools seemed to be
b. Be arranged to allow wants to equip his failing? In their obligation
students to receive students with the skill to to:
feedback ***** organize information a. Respect for all duly
c. Take place over a long gathered? constituted authorities
period of time a. Note-taking, outlining, b. Instill allegiance to the
d. Be difficult enough for using the library Constitution
students to remember b. Outlining, c. Promote obedience to
and learn summarizing, using the the laws of the state
459. Which of the card catalogue d. Promote national pride
following is a practice in c. Note-taking, outlining, *****
progressive education? summarizing ***** 464. To be an effective
a. Open classroom d. Summarizing, note classroom manager,
***** taking using the library teachers must be friendly
b. Study of classics 462. Annual medical but at the same time be:
c. Academic orientation checkup required of a. Buddy-buddy
d. Cognitive education teachers is done in the b. Rigid
460. The workers’ rights interest of: c. Business-like *****
d. Highly demanding explore their feelings and brings students out of the
465. Which of the ideas***** classroom, let us bring it
following field of Social c. Come up with highly into the classroom.
Sciences below is more competitive games where Perhaps, I can use it to
connected with the study winners will feel happy teach Math.” To which
of social traditions and d. Establish the disciple philosophy does Teacher
cultures? of being judgmental in Benny adhere?
a. Theology attitude a. Progressivism c.
b. Psychology 468. Which statement Essentialism
c. Sociology ***** holds true to grades? b. Existentialism ***** d.
d. Anthropology Grades _________. Reconstructionism
466. When an individual a. are exact measure of 471. “The greatest
or group adapts the IQ and achievement happiness lies in the
culture of others, practice b. are a measure of contemplative use of the
them and become achievement ***** mind”, said Plato.
habitual, this is: c. are necessarily a Therefore, let us give
a. Culture change ***** c. measure of students’ IQ more opportunities for our
Culture shock d. are intrinsic motivation students to do
b. Culture lag d. Culture for learning __________.
difference 469. Which practice does a. Social interaction c.
467. Which appropriate not fit in a classroom that Role playing
teaching practice flows recognizes individual b. Introspection d.
this research finding on differences? Cooperative learning
the brain: “The brain’s a. Uniform requirements 472. Why is it sound to
emotional center is tied ***** encourage students to
into its ability to learn.” b. Sharing from multiple define terms in their own
a. Tell the students to perspective words? Because
participate in class c. Accommodating ____________.
activities or else won’t student’s learning styles a. Defining the terms in
receive plus points d. Various modes of their own words helps
b. Create a learning assessing learning them memorize the
environment that 470. Teacher Benny definition faster
encourages students to says: “If it is billiard that b. Students remember
information better when d. Accommodation guiding principle?
they mentally process in problem a. I must teach the child
some way 475. Were teachers in the so he is assured of
c. They ought to connect Philippines required of a heaven
the terms that they learn professional license since b. I must teach the child
with other terms the establishment of the to develop his mental
d. This is one opportunity Philippine educational powers to the fullest *****
to brush up with other system? c. I must teach the child
terms a. No, but the equivalent that we can never have
473. Pavlov is to classical of a license required was real knowledge of
conditioning as ______ is a certificate in teaching anything
to operant conditioning. b. Yes, it was required d. I must teach the child
a. A. Bandura since the Americans every knowledge, skill
c. J. Watson established the and value that needs for
b. J. Holt d. B.F. educational system a better future
Skinner ***** c. No, it was only with the 478. A teacher put
474. You arrange the effectivity of R.A. 7836 together the output of her
rows of blocks in such a that professional license colleagues in one
way that a row of 5 was required ***** workshop and published
blocks is longer than a d. Yes, except for the it with her name as
row of 7 blocks. If you Thomasites author. Which is
ask which row has more, 476. The increase in the unprofessional about the
Grade 1 pupils will likely number of school children teacher’s behavior?
say that it is the row that left by OFW parents a. Failing to correct what
makes the longer line. intensifies the teacher appears to be
Based on Piaget’s role as ______________. unprofessional conduct
cognitive development a. Student’s friends b. Giving due credit to
theory, what problem is b. Guidance counselors others of their work
illustrated? c. Facilitator of learning c. Not giving due credit to
a. Conservation problem d. Substitute parents ***** others for their work *****
***** 477. As a teacher, you d. Holding inviolate all
b. Assimilation problem are a rationalist. Which confidential information
c. Egocentrism problem among these will be your concerning associates
479. In what way can which particular skills his 484. The Thematic
teachers uphold the pupils are weak; which Appreciation Test is an
highest possible test should Teacher Neil example of a (an)
standards of the teaching give? __________.
profession? a. Standard Test c. a. Self-report technique
a. By pointing out the Diagnostic Test***** b. Projective technique
advantages of joining the b. Placement Test d. *****
teaching profession Aptitude Test c. Interest inventory
b. By good grooming to 482. In the context of d. Socio-metric technique
change people’s multiple intelligences,
perception of teacher which one is the 485. The following are
c. By continuously weakness of the paper- features of the
improving themselves pencil test? Restructure Basic
personally and a. It put non-linguistically Education Curriculum,
professionally ***** intelligent pupils at a except:
d. None of the above disadvantage ***** a. Increased time for
480. To reach out to b. It requires paper and tasks to gain mastery of
clientele who cannot be printing and is so competencies
in the classroom for one expensive b. Interdisciplinary modes
reason or another, which c. It utilizes so much time of teaching
of the following was d. It lacks reliability c. Greater emphasis on
established? 483. Out of 3 distracters content, less on the
a. Special education in a multiple choice test learning process*****
(SPED) item, namely X, Y and Z, d. Stronger integration of
b. Informal education no pupil chose Z as an competencies and
c. Alternative learning answer. This implies that values, across the
delivery system***** Z is ________. learning area
d. Pre-school education a. An effective distracter
481. Teacher Neil b. A plausible distracter 486. The free public
discovered that his pupils c. A vague distracter elementary and
are weak in d. An ineffective secondary education in
comprehension. To distracter***** the country is in the line
further determine in with the government
effort to address observes cleanliness and assessment tool for an
educational problems of order in her classroom to instructional objective on
_______. create a conductive working with and relating
a. access and equity ***** atmosphere for learning. to people?
b. relevance and quality On which theory is her a. Writing articles on
c. effectiveness and practice based? working and relating to
efficiency a. Behaviorism ***** people
d. productivity b. Psychoanalysis b. Organizing a
487. The task of setting c. Gestalt psychology community project *****
up routine activities for d. Humanistic psychology c. Home visitation
effective classroom 490. Which activity is d. Conducting mock
management as a task meant for kinesthetically election
that a teacher should intelligent pupils?
undertake ______. a. Independent study 493. Which physical
a. on the very first day of b. Individualized study arrangement of chairs
school ***** c. Pantomime ***** contributes to effective
b. every day at the start d. Cooperative learning classroom management?
of the session 491. With which will the a. Sticks to the traditional
c. every homeroom day existentialist agree? The chair arrangement in the
d. as soon as the school is a place where classroom
students have adjusted individuals _____. b. Distinguishes teacher
on their schedule a. Listen and accept what from students
488. Teacher Honey uses the teacher say c. Makes it easier to
direct instruction strategy. b. Can meet to pursue clean the room
Which will she first do? dialogue and discussion d. Enhances classroom
a. Independent practice about their lives and interaction *****
b. Guided student choices*****
practice c. Can observe by using 494. Each teacher said to
c. Review the previous their senses to the be a trustee of cultural
day’s work ***** maximum and educational heritage
d. Presenting and d. Can reflect on ideas of the nation and under
structuring 492. Which will be the obligation to transmit to
489. Teacher Janice most authentic learners such heritage.
Which practice makes Diary b. Rationalism d.
him fulfill such obligation? b. Observation report d. Impersonalism
a. Use of the latest Portfolio *****
instructional technology 498. To develop 501. Conducting follow
b. Study of the life of reasoning and speaking up studies of graduates
Filipino heroes ***** ability, which should I and drop out is a
c. Use of interactive use? guidance service that
teaching strategies a. Debate***** c. falls under:
d. Observing continuing Experiment a. Placement services
professional education b. Storytelling d. Role- b. Research service*****
495. Writing an original playing c. Individual inventory
essay is an example of 499. Under which type of services
which level of objective in guidance service does d. Counseling service
the cognitive domain? the concern of schools to
a. Evaluation c. Analysis put students into their
b. Synthesis d. most appropriate courses 502. Under which
Application ***** fall? assumption is portfolio
a. Individual inventory assessment based?
496. Which terms refers service a. Assessment should
to a teacher helping a b. Research service stress the reproduction of
colleague grow c. Placement service***** knowledge
professionally? d. Information service b. An individual learner is
a. Technology transfer 500. After having been adequately characterized
b. Independent study humiliated by his teacher, by a test score
c. Facilitating a student evaluates that c. An individual learner is
d. Peer mentoring ***** teacher very poorly, inadequately
497. Which terms refers despite teacher’s characterized by a test
to the collection of excellent performance. score
student’s products and Which trait is illustrated d. Portfolio assessment is
accomplishments for a by the student’s dynamic assessment
period for evaluation behavior? *****
purposes? a. Particularism c. 503. Which program was
a. Anecdotal record c. Personalism***** adopted to provide
universal access to basic campus expression so.” Based on Kohlberg’s
education to eradicate “promdi” and barriotic” theory, in which moral
illiteracy? indicate? development stage is
a. Values educational a. The powerlessness of Billy?
framework the poor a. Pre-Conventional level
b. Education for all ***** b. The power of the rich b. Conventional level
c. “Paaralan sa bawat c. Low literacy rate of the *****
barangay” country c. Between conventional
d. Science and Education d. The prevalence of and post conventional
Development Plan ethnocentrism ***** levels
504. Which schools are 507. Why is babyhood d. Post-Conventional
subject to supervision, referred to as a “critical level
regulation and control by period” in personally 509. If you plan to
the state? development? Because develop a lesson on
a. Public, Private _________. using s-verb with the third
sectarian and Non- a. The foundation is laid person singular as
sectarian ***** upon in which the adult subject deductively, what
b. Sectarian and non- personally structure will is the first step in your
sectarian school be built***** lesson development
c. Private school b. The baby is exposed to outline?
d. Public schools many physical and a. Give sentences using
505. Here is a score psychological hazards s-verb form
distribution: 98, 93, 93, c. The brain grows and b. Ask the students about
93, 90, 88, 87, 85, 85, 85, develops as such s-verb form and third
70, 51, 34, 34, 34, 20, 18, accelerated rate during person singular as
15, 12, 9, 8, 6, 3, 1. What babyhood subject
is the characteristic of the d. Changes in the c. State the rule on
score distribution? personality pattern take subject-verb agreement
a. Bimodal place for third person as
b. Trimodal***** 508. Billy, a grade 1 pupil subject *****
c. Skewed to the right is asked, “Why do you d. Conduct appropriate
d. No discernible pattern pray every day?” Billy sentence drill
506. What do the school answer. “Mommy said 510. For lesson clarity
and effective retention, while the rest scored 45 d. The bell curve shape is
which one should a and above. This means steep
teacher observes, that Teacher Edna 515. Jan, a grade 1 pupil,
according to Bruner’s ________. is happy when he wins a
theory? a. Did not attain her game but skulks when he
a. Start at the concrete lesson objective because doesn’t. Which concept
level and end there of the pupil’s lack of does his behavior
b. Begin teaching at the attention indicate?
concrete level but go b. Attained her lesson a. Egotism c.
beyond it by reaching the objective because of the Semi-logical reasoning
abstract ***** pupil’s lack of attention b. Egocentrism***** d.
c. End teaching with c. Attained her lesson Rigidity of thought
verbal symbol objective 516. The practice of non-
d. Use purely verbal d. Failed to attain her graded instruction stems
symbols in teaching lesson objective as far as from ________.
511. Which activity the 25 pupils are a. Progressivism***** c.
should a teacher have concerned ***** Existentialism
more for his students if 513. With assessment of b. Reconstructionism d.
he wants them to develop affective learning in mind, Essentialism
logical-critical thinking? which does not belong to 517. Theft of school
a. Symposium c. the group? equipment like TV,
Brainstorming a. Moral dilemma c. computer, etc. by
b. Debate ***** d. Panel Diary entry teenagers in the
discussion b. Reflective writing d. community itself is
512. The criterion of Cloze test***** becoming a common
success in Teacher 514. Which is true when phenomenon. What does
Edna’s objective is that standard deviation is big? this incident signify?
“the pupils must be able a. Scores are a. Deprivation of Filipino
to spell 90% of the words concentrated schools
correctly.” Linda and b. Scores are not b. Inability of school to
other 24 students in the extremes hire security guards
class spelled only 40 out c. Scores are spread c. Prevalence of poverty
of 50 words correctly apart***** in the community
d. Community’s lack of capacity. What does this b. Learning is defined as
sense of co-ownership imply? a change in the learner’s
***** a. Every child is a observable performance
518. What does extreme potential genius ***** *****
authoritarianism in the b. Pupils can possibly c. The success of
home reinforce in reach a point where they learners is based on
learners? have learned everything teacher performance
a. Creativity in work c. Some pupils are d. Not every form of
b. Ability to direct admitted not capable of learning is observable
themselves learning 523. The following are
c. Doing things on their d. Every pupil has his its used in writing
own initiative own native ability and his performance objective,
d. Dependence on others learning is limited to this except?
for direction ***** native ability a. Integrate c. Diagram
519. The main purpose of 521. The principle of b. Delineate d.
compulsory study of the individual differences Comprehend*****
constitution is to requires teachers to 524. Which is/are sign/s
________. _____. of the student with
a. Make constitutional a. Treat all learners alike Attention Deficit
experts of the students while in the classroom Disorder?
b. Develop students into b. Prepare modules for a. Impatient while waiting
responsible, thinking slow learners in class for his/her turn during
citizens ***** c. Give greater attention games *****
c. Prepare students for to gifted learners b. Completes work before
law-making d. Provide for a variety of shifting to another
d. Acquaint students with learning activities***** c. Excessively quiet
the historical 522. Which assumption d. Cares for his/her
development of the underlines the teacher’s personal things
Philippine Constitution use of performance 525. You observe that
520. Studies in the areas objectives? pupils answer even when
of neurosciences a. Performance not called, shouts MA’AM
disclosed that the human objectives assure the to get your attention, and
brain has limitless learner of learning laugh when someone
commits mistakes. What arrangement has been b. Realia d. Chart
should you do? proven to be effective for
a. Send the misbehaving learning? 531. Teacher Bonnie
pupils to the guidance a. Flexible to suit varied likes to concretize
counselor activities abstract concept of the
b. Set the rules for the b. Fixed arrangement to water molecule. She
class to observe maximize instructional came up with a concrete
c. Involve the whole class time presentation by using
in setting rules of conduct c. Any seat arrangement wires and plastic balls.
for the whole class***** to suit varied learning How would you classify
d. Make a report to the styles***** Teacher Bonnie’s visual
parents about their d. A combination of fixed aid?
children’s misbehavior and flexible arrangement a. Replica c. Realia
526. Which is the final, 529. Which can run b. Chart d. Mock-up
indispensable component counter to the *****
of a lesson plan? encouragement you give 532. Teacher Lenny
a. Evaluation ***** c. to your students to ask demonstrated to the class
References questions? how to focus the
b. Activity d. a. Eye to eye contact microscope, after which
Assignments b. An encouraging hand the students were asked
527. Carlo, a grade 2 gesture to practice. Which
pupil, plays with his c. Radiant face teacher prompting is least
classmates but cannot d. Knitted eyebrows when intrusive? Teacher Lenny
accept defeat. Based on a question is raised ***** ________.
Piaget’s theory on a. Held the hand of a
cognitive development, in 530. Teacher Agot likes student and with her
what development stage to show how the hand holding the
is Carlo? launching of spaceships student’s hand adjusted
a. Formal operation c. takes place. Which of the the mirror
Pre-operational ***** following materials b. Pointed to the mirror
b. Concrete operation available is most fit? and made an adjusting
d. Sensorimotor a. Mock-up***** c. gesture with her hand
528. Which seating Replica c. Adjusted the mirror
d. Reminded the class to b. Increase learning c. Thematic tests
first adjust the mirror ***** c. Speed learning d. Stylistic tests
d. More interesting
533. A pupil who has learning 539. Which process
developed a love for 536. Which refers to the enhances the
reading keeps in reading Filipino trait of practicing comparability of grades?
for his enjoyment. His conflicting values in a. Giving more Higher
motivation for reading is: different venues and with Order thinking skills
a. Insufficient c. Extrinsic different social groups? b. Constructing
b. Intrinsic d. Both a. “Kanya-kanya” departmentalized exam
intrinsic and extrinsic***** mentality ***** for each subject area*****
534. Which educational b. Procrastination c. Determining the level
trend is occurring in all c. Existential intelligence of difficulty of the test
modern societies as a d. Crab mentality d. Using a table of
result of knowledge 537. Which of these can specification
explosion and rapid measure awareness of
social, technological and values? 540. Under which type of
economic changes? a. Projective techniques guidance service does
a. Nuclear education b. Rating scales the concern of school to
b. International education c. Moral dilemmas ***** put students into their
***** d. Sociogram most appropriate courses
c. Lifelong learning 538. Which test fall?
d. Team teaching determines whether a. Information service
students accept b. Placement service
535. With the advent of responsibility for their *****
multi-media resources own behavior or pass on c. Individual inventory
and computers, which is responsibility for their services
the most favorable result own behavior to other d. Research services
of the optimal use of people? 541. The teacher’s role in
educational media a. Locus-of-control tests the classroom according
technology? ***** to cognitive psychologist
a. Interactive b. Sentence-completion is to _______.
learning***** tests a. Make the learning task
easy for the learner d. The bell curve shape is killed is a stranger, not in
b. Dictate what to learn steep. any way related to him
upon the learner 544. Here is a test item: 547. A political boss
c. Fill the minds of the DISTANT : NEAR :: builds a school in a
learner with information GENUINE : _______. distant barrio in order to
d. Help the learner This item is a/an get the votes for an
connect what they know _________. unworthy and corrupt
with new information from a. Analogy***** c. candidate. Is the action of
the teacher***** Metaphor the political boss moral?
b. Riddle d. Completion a. No, the candidate is
542. Which questioning 545. What refers to a undeserving
technique would be single word or phrase b. No, his move was not
appropriate for inductive that tells the computer to meant for good effect
lessons? do something with *****
a. Involve students program or file? c. Yes, the votes were
actively in the questioning a. Computer language exchange for the school
process***** c. Command ***** built
b. Expect participation b. Computer program d. d. Yes, it was his duty to
only among the more Password strategies for his
motivated students 546. Can an insane candidate to win
c. Use questions person be blamed for 548. The teacher’s first
requiring only memory killing a stranger? task in the selection of
responses a. Yes, because an media in teaching is to
d. As a teacher, you ask insane person possesses determine the:
no questions a little degree of a. Choice of the teacher
543. Which statement is voluntariness b. Availability of the
true in a bell-shaped b. Yes, because an media *****
curve? insane person is not c. Technique to be used
a. There are more high totally ignorant d. Choice of the students
scores than low scores c. No, because of his 549. Which is the true
b. Most scores are high ignorance and lack of foundation of the social
c. The scores are voluntariness***** order?
normally distributed***** d. No, because the one a. Strong political
leadership 553. The following are school
b. The reciprocation of trends in marking and 556. In what way can
rights and duties ***** reporting system, except: instructional aides
c. Equitable distribution of a. Supplementing subject enhance learning?
wealth grades with checklist on a. Entertain student
d. Obedient citizen traits b. Hold students in the
550. All the examinees b. Conducting parent- classroom
obtained scores below teacher conferences as c. Reinforce
the mean. A graphic often as needed learning*****
representation of the c. Raising the passing d. Take the place of the
score distribution will be: grade from 70 to 80 ***** teacher
a. Perfect normal curve d. Indicating strong points 557. The study on types
b. Negatively skewed as well as those needing of reading exercises
c. Positively skewed improvement gives practice in:
d. Leptokurtic 554. Which is a type of a. All sorts of study
551. In a normal graph in which lines methods *****
distribution curve, a T- represent each score or b. Reading skills needed
score of 70 is: set of scores? in other project
a. Two SDs above the a. Histogram c. Recognizing the
mean ***** b. Scatter gram precise meaning of words
b. Two SDs below the c. Scatter plot d. Picking out the man
mean d. Frequency polygon ideas
c. One SD below the *****
mean 555. Which is an example 558. How students learn
d. One SD above the of a perfect duty? may be more important
mean a. Paying the worker the than what they learn.
552. Which one stifles wages agreed upon ***** From this principle, which
student’s initiative? b. Donating an amount of the following is
a. “Bahala na” for a noble project particularly important?
b. “Utang na loob” c. Giving alms to the a. Knowing how to solve
c. Rationalism needy a problem *****
d. Extreme d. Supporting a poor but b. Solving a problem
authoritarianism ***** deserving student to within time allotted
c. Getting the right in place of a friend 565. The military training
answer to a word c. We charge to requirements among
problem experience our being students in the secondary
d. Determining the given punished and tertiary levels can be
559. Kiko is very attached d. See someone get traced as a strong
to his mother and Sharon punished for habitual influence of the:
to her father. In what tardiness. In effect, we a. Greeks ***** c.
developmental stage are are less likely to be Chinese
they according to tardy***** b. Romans d. Athenians
Freudian’s psychological 562. In instructional 566. Teacher Marissa
theory? planning, it is necessary wants to review and
a. Phallic stage***** c. that the parts of the plan check on the lesson of
Oedipal stage from the first to the last the previous day? Which
b. Latent stage d. Anal have: one will be most reliable?
stage a. Symmetry c. a. Having students
560. A student dislikes Conciseness correct each other’s work
Math due to traumatic b. Coherence ***** d. b. Having students
experience in the past. Clarity identify difficult homework
Which law explains this? 563. If Teacher Judith problems
a. Partial activity c. has to ask more higher- c. Explicitly reviewing
Vividness order questions, she has the task relevant
b. Analogy d. to ask more ________ information for the day’s
Disposition/Mind set ***** questions. lesson *****
a. convergent c. fact d. Sampling the
561. Which illustrates b. closed d. understanding of a few
vicarious punishment? divergent***** students
a. We feel so bad to a 564. For maximum 567. To promote effective
classmate who is interaction, a teacher practice, which guideline
punished for being tardy ought to avoid ______ should you bear in mind?
so we convince him go to questions Practice should be ____.
school on time a. rhetorical c. divergent a. Difficult for students to
b. Out of comparison, we b. leading ***** d. learn a lesson
volunteer to get punished informational b. Arranged to allow
students to receive Whole-Part-Whole ***** at once remarked.
feedback ***** 570. For which may you “What’s wrong?” What
c. Done in an evaluative use the direct instruction does this imply?
atmosphere method? a. Reporting to a
d. Take place over a a. Use a microscope Guidance Office is often
long period of time properly ***** associated with
568. Which is one role of b. Distinguish war from misbehavior*****
play in the pre-school and aggression b. Student Ben is a
early childhood years? c. Appreciate Milton’s “problem” student
a. Separates reality from Paradise Lost c. Guidance counselors
fantasy d. Become aware of the are perceived to be
b. Develops the upper pollutants around us “almighty and omniscient”
and lower limbs 571. By what name is d. The parents of
c. Develop competitive indirect instruction or Students Ben must be of
spirit Socratic methods also the delinquent type
d. Increase imagination known? 574. A teacher combined
due to expanding a. Questioning method several subject areas in
knowledge and emotional ***** order to focus on a single
range***** b. Morrison method concept for
569. Teacher Joel taught c. Indirect method interdisciplinary teaching.
a lesson denoting d. Mastery learning Which strategy/method
ownership by means of 572. Which does not did he use?
possessives. He first belong to the group of a. Unit method
introduced the rule, then alternative learning b. Thematic instruction
gave examples, followed systems? *****
by class exercise, then a. Multi-age grouping c. Problem entered
back to the rule before he b. Multi-grade grouping learning
moved it the second rule. c. Non-graded grouping d. Reading-writing
Which presenting d. Graded education ***** method
technique did he use? 573. Student Ben was 575. Which is a major
a. Sequential c. asked to report to the advantage of curriculum-
Comparative Guidance Office. Student based assessment?
b. Combinational d. Ben and his classmates a. It tends to focus on
anecdotal information on opposition from the as a planet*****
student’s progress persons themselves. ***** 580. In which of the
b. It is based on a norm 577. In the problem following would
referenced measurement solving method of programmed learning to
model teaching, which is the be most likely to be
c. It is informal in nature primary role of the found?
d. It connects testing with teacher? a. In a class divided into
teaching***** a. Clarifier ***** c. small groups
576. A school Division Observer b. Independent study
Superintendent was b. Judge d. Director c. In dyadic groups
enthusiastically lecturing 578. To educate the child d. In a class where
on the Accreditation for freedom is to educate teacher tries to
Program for Public him/her to: individualized instruction
Elementary Schools I. Respond to others *****
(APPES), its benefits and II. Respond to 581. A master teacher,
demands. Thinking of its himself/herself the resource speaker in
many demands, most of III. Do as he/she pleases an in-service training,
the school heads were IV. Live as he/she desires presented the situated
not very happy about it a. II and III c. I and II learning theory and
and the older one were ***** encouraged her
whispering “we have very b. I and III d. I and IV colleagues to apply the
reason to retire soon.” 579. I want to engage my same in class. Which did
What does this tell about students in small group she not encourage her
the change process? discussion. Which topic colleagues to do?
a. People resist change lends itself to a lively a. Apprenticeship
for no reason discussion? b. Learning as it normally
b. People tend to resist a. The meaning of the occurs
change law of supply and c. Authentic problem
c. Resistance to change demand solving
is insurmountable b. Rules on subject-verb d. Decontextualized
d. Leadership can affect agreement teaching *****
the desired change in c. The law of inertia 582. Which is the ultimate
persons despite d. The exclusion of Pluto aim of classroom
management? valid indicators of to peace education?
a. To set up condition students’ achievements. a. Learning to do
that brings about effective Which process should be b. Learning to know
teaching and learning observed? c. Learning to live *****
***** a. Explaining the d. Learning to be
b. To secure conformity meaning of marks and
to rules with ease grades 588. The failure of
c. To make children b. Defining the course independent study with
realize that they cannot objective as intended most Filipino students
do everything they want learning outcomes ***** may be attributed to
d. To remove the physical c. Adopting letter grades students’ ______.
condition in the room such as A, B, C and D a. high degree of
583. Under which d. Giving objective type of independence
teaching strategy does a tests b. ambiance
School’s division practice 586. Who are not c. unpreparedness for
of assigning a Girl Scout covered by the Code of schooling *****
to serve as Ethics of Professional d. high degree of
Superintendent of the Teachers? dependence on authority
Day or Mayor of the Day a. All full time or part time 589. When you use the
for leadership training public and private school overhead projector for
fall? teacher and administrator topic presentation, point
a. Panel discussion b. Teachers of academic, to the ______.
b. Symposium vocational, special, a. OHP slide c. OHP
c. Simulation ***** technical or non-formal screen
d. Dramatization institution b. OHP light d.
584. Which approach c. Teacher in the tertiary Projection wall *****
makes you think of your level ***** 590. Where do you make
thinking? d. Teacher in all the correction of your
a. Constructivist c. educational institutions at notes while using the
Cognitive all levels overhead projector?
b. Metacognitive d. 587. Which of the a. On the slide *****
Integrative UNESCO’s four pillars of b. On the overhead
585. The grades make education is most related projector
c. On the projector wall b. Existentialism d. this?
d. On the screen Progressivism a. Classical conditioning
594. Jonna, a principal, *****
591. Authority comes shares this thought with b. Generalization
from God and is meant her teachers. “Subject c. Operant conditioning
to: matter should help d. Attribution theory
a. Help those given the students understand and 597. One strength of an
authority to do their task appreciate themselves as autobiography as a
***** unique individuals who technique for personality
b. Distinguish those with accept complete appraisal is that
authority from those responsibility for their ________.
without thoughts, feelings and a. It may be read by
c. Be lorded over others action.” From which unauthorized people
d. Make the subjects of philosophy is this though b. It can replace data
authority recognize their based? obtained from other
superiors a. Essentialism c. data-gathering technique
592. For a discussion of a Progressivism c. It makes possible the
topic from various b. Perennialism d. presentation of intimate
perspectives, it is best to Existentialism ***** experiences *****
hold a/an ______. 595. Who stressed the d. It gives complete data
a. Brainstorming c. idea that students cannot about the author
Debate learn if their basic needs 598. Which Millennium
b. Symposium d. Panel are not first met? Development Goal
discussion***** a. Maslow***** c. (MDG) goal is related to
Wertheimer the state’s goal for quality
593. A Principal tells her b. Miller d. Thorndike education?
teacher that training in 596. A person, who has a. 1 b. 2 c. 3 d. 4
the humanities is most had painful experiences 599. All of the following
important. To which at the dentist’s office, describe the development
educational philosophy may become fearful at of children aged eleven to
does he adhere? the mere sight of the thirteen, except:
a. Perennialism ***** c. dentist’s office building. a. Sex differences in IQ
Essentialism Which theory can explain become more evident
***** lecture on fossils given by of other people
b. They exhibit increased the teacher c. He interprets events
objectivity in thinking b. The child goes out and form a limited views
c. They shift from discovers for himself d. He sees events apart
impulsivity to adaptive some rock or fossil ***** from himself and other
ability c. The child summarize people *****
d. They show abstract the section on fossils in 605. Who introduced the
thinking and judgment his science textbook technique of using the
600. One learns Math by d. The child copies a list drawing of a man as a
building on the Math of facts concerning fossils measure of intelligence?
concepts previously on the blackboard a. Aristotle c.
learned. This is an 603. Social development Goodenough*****
application of: means the acquisition of b. Herbert d. Binet
a. Constructivist***** c. the ability to behave in
Physiological accordance with: 606. Which Republic Act
b. Humanist d. S-R a. Stereotyped behavior provides government
b. Social expectation assistance to students
601. The singing of the ***** and teachers in private
National Anthem in c. Social insight education?
schools is an offshoot of d. Universal norms a. RA 7784 c. RA 7836
the philosophy of: b. RA 6728***** d. RA
a. Nationalism ***** c. 604. When an adolescent 6675
Naturalism combines ability to use
b. Pragmatism d. deductive and inductive 607. The authoritarian
Socialism reasoning in constructing setting in the Filipino
realistic rules that he can home is reinforced by a
602. The environment in respect and live by, how classroom teacher who:
order to facilitate learning does he perceive his a. Is open to suggestions
must be interactive. environment? b. Encourage pupils to
Which of the following a. He views the world ask questions
best typifies this kind of from his own perspective c. Prescribes what pupils
environment? b. He sees the world and should do *****
a. The child listens to a himself through the eyes d. Ask open ended
questions the detriment of pupils’ 612. In which way does
performance illustrates heredity affect the
608. Who among the the: development of the
following believes that a. Filipino’s lack of learner?
learning requires seriousness a. By placing limits
disciplined attention, b. Filipino’s love for beyond which the learner
regular homework, and “porma” ***** cannot develop *****
respect for legitimate c. Filipino’s lack of b. By providing equal
authority? reflection potential to all
a. Essentialist***** c. d. Filipino’s sense of c. By compensating for
Progressivist humor what environment fails to
b. Perennialist d. develop
Reconstructionist 611. In order to avoid d. By blocking the
609. The Constitutional disgrace, a pregnant, influence of environment
provision on language unmarried woman takes 613. The cultivation of
has the following aim, drugs to induce abortion. reflective and meditative
except: Is she morally justified to skills in teaching is an
a. To make Filipino the do that? influence of:
sole medium of a. Yes, it can save her a. Taoism c.
instruction***** and child from disgrace Confucianism
b. To make the regional when he grows up b. Shintoism d. Zed
dialect as auxiliary media b. No, the act of inducing Buddhism *****
of instructions in regional abortion is bad in itself
school c. No, the unborn child 614. A child refuse to
c. To maintain English as cannot be made to suffer obey orders or displays
a second language the consequences of the negativism as a
d. To make Filipino the sins of his parents ***** development trait. How
national Language and d. No, it is better to may you best handle
medium of instruction and prevent the child from him?
communication coming into the world a. Detain him after office
610. The tendency to who will suffer very much hours for him do to what
emphasize so much on due to the absence of a he has been ordered to
school beautification to father do
b. Take every opportunity pangako ng anak. 620. Which of the
to praise him for every a. Sumasagi ***** c. following is usually
positive attitude display Bumubuhay considered the most
***** b. Gumugulo d. important factor in a
c. Insist on compliance to Sumasapi child’s observable
the same degree required 618. Huwag kang classroom behavior?
of pupils maniniwala sa bulaklak a. Intelligence c. Self
d. Avoid giving him ng kanyang matamis na concept *****
orders if you do and he dila: b. Heredity d. Cultural
objects take back the a. Ito’y panunukso background
order b. Ito’y pambobola *****
615. Which term refers to c. Ito’y pagbibiro 621. Section 5, Article
the collection of students’ d. Ito’y pagsisinungaling XIV, of the Constitution
products and states that academic
accomplishment for a 619. If a resilient child freedom shall be enjoyed
period of evaluation with superior intelligence in:
purposes? is reared in a poor a. Public assemblies
a. Portfolio***** c. environment the probable b. All institution of higher
Anecdotal record outcome would be: learning *****
b. Observation report d. a. No change in IQ c. State colleges and
Diary because environment universities
deprivation has nothing to d. All levels of learning
616. For comparing and do with intelligence
contrasting which graphic b. Mental retardation 622. A teacher who
organizers is most since he is culturally subscribes to the
appropriate? deprived pragmatic philosophy of
a. Cycle c. Story map c. Slight change in IQ education believes that
b. Web d. Venn although he can experience should follow
Diagram***** overcome frustration and learning in her teaching,
obstacle ***** she therefore exerts effort
617. Laging d. Great change in IQ in:
UMUUKILKIL sa isipan because he is culturally a. Encouraging learners
ng ama ang nasirang deprived to memorize factual
knowledge the school canteen even person strive to meet his
b. Providing learners if you cannot afford to do needs?
opportunities to apply every day a. Minimize the
theories and principles 624. How can you help a unpleasant
***** habitual borrower of consequences of student
c. Equipping learners with money get rid of his involvement *****
the basic abilities and habit? b. Utilize your own
skills a. Let him do something opinion as teacher in
d. Requiring learners full for you in return for the making final decisions in
mastery of the lesson money you lent him the classroom
b. Direct him to others c. Use unfamiliar
623. As a parent and at c. Do not lend him materials as examples in
the same time a teacher, anymore ***** order to initially arouse
which of the following will d. Ask for a collateral for their curiosity
you do to show your the cash he is loaning d. Ask pupils to submit
cooperation to a PTA 625. Periodic checks on test questions or
project in your school to student seatwork with a reactions which you can
be financed with the smile and pat on the select topics
proceeds of the sales of shoulder effectively 627. Which is not a
the school canteen where reinforce good study characteristic of a
food prices are little bit habit is an example of: democratic discipline?
higher? a. Discrimination a. Child has opportunity
a. Bring food for you and reinforcement to expense his/her
your children, but always b. Variable-ratio schedule opinion
make it a point to buy in c. Continuous b. Child’s given
the school canteen***** reinforcement ***** punishment is related to
b. Buy all your food in the d. Fixed interval and the misdeed
school canteen but variable-interval schedule c. Child understands the
request for a discount 626. A person strives to meaning of rules
c. Bring food enough for work at a given task d. Child obeys blindly
you and your children but because of a need. *****
do not eat in the canteen Which of the following 628. Who among the
d. Buy all your food from situations can make a following stressed the
processes of experience of work ***** b. Exercise ***** d.
and problem solving? 631. When a school Belongingness
a. Dewey ***** c. Hegel decides to work on a 635. Positive
b. Aristotle d. Plato thematic curriculum interdependence as an
629. Which of the which should be out of element of collaborative
following reasons of the picture? learning means that the
measuring student a. Peer collaboration students must:
achievement is not valid? b. Integration a. Learn to depend on
a. To prepare feedback c. Team teaching each other to achieve a
on the effectiveness of d. Competition ***** goal *****
the learning process 632. Teacher Mary wants b. Depend on the diligent
b. To certify that students to teach her pupils the students
have attained a level of technique on reading for c. Help one another in the
competence in a subject information. Which individual test for
area technique should be everyone to pass
c. To discourage students used? d. Be grouped
from cheating during test a. Text structure c. Story heterogeneously
and getting high map
scores***** b. Prior knowledge d. 636. Which computer
d. To motivate students SQ3R***** seems to have the most
to learn and master the 633. In instructional potential for the
materials they think will planning, which among classroom?
be covered by the these three: unit plan, a. Mainframe computer
achievement test course plan, lesson plan b. Minicomputer
630. Which characterizes is (are) most specific? c. Microcomputer *****
the perfectionist type of _________ plans. d. LPC
students? a. Course and lesson c.
a. Does not volunteer or Lesson ***** 637. A teacher notices
initiate b. Course d. Unit glaring wrong
b. Give up easily 634. The use of drills in pronunciation of vowel
c. Rarely complete tasks the classroom is rooted sounds among her
d. Often anxious, fearful on Thorndike’s law of: students necessitating
or frustrated about quality a. Readiness c. Effect more practice. Which of
the following activities carry the crosses 643. Rights which cannot
would be most helpful? imposed can share the be renounced or
a. Dictionary use c. joy of life***** transferred because they
Assignments c. The more suffering in are necessary for the
b. Review d. Drill ***** this life, the more one is fulfillment of man’s
assured of heaven primordial obligations are
638. What storage device d. One should look for called:
is significantly more suffering to save a. Alienable rights c.
efficient in holding himself/herself Inalienable rights*****
information? 641. Which trust on value b. Perfect rights d.
a. Hard disk ***** c. formation is meant to Acquired rights
Floppy disk help the students make 644. Which is in line with
b. Software d. Audio use of their thinking and equitable access to
cassette scientific investigation to education but runs
639. The Filipino decide on topics and counter to quality?
tendency to resort to the questions above values? a. Selective retention of
easy way out from a term a. Value inculcation c. students
paper as a course Value clarification***** b. Deregulated tuition fee
requirement by hiring a b. Analysis d. hike
ghost writer or by passing Moral development c. Open admission *****
a photocopied term paper d. Program accreditation
provide which Filipino 642. Which interactive 645. A negative
traits? teaching should be discrimination index
a. Anticipation c. avoided? means that:
Pakikisama a. Using multiple a. The items could not
b. Ambivalence d. Lack response strategy discriminate between the
of discipline***** b. Using “put down” lower and upper group
640. “No pain, no gain.” strategy ***** b. More from the lower
This means that: c. Asking more divergent group answered the test
a. One should be penitent questions items correctly *****
every Friday by carrying d. Asking more evaluative c. More from the upper
his cross questions group answered the test
b. Only those willing to item correctly
d. Less from the lower learning teachers and students to
group got the test item b. Massed learning is as hold a demonstration to
correctly effective as spaced oust the superior
learning
646. Your teacher is of c. Spaced learning is 650. Pick out the situation
the opinion that the world better than massed that illustrates the duty of
and everything in it are learning ***** a new teacher to the
ever changing and so d. Both massed learning state:
teaches you the skill to and spaced learning are a. Take a long vacation
cope with the changes. not effective which she firmly believes
What is his governing she deserves after four
philosophy? 649. Which of the years of diligent study
a. Experimentation c. following measures before taking the
Realism should a teacher do to a examination for teachers
b. Existentialism ***** d. principal whom she would b. Take the licensure
Idealism like to file a case of examination for teacher
sexual harassment and an oath to do her
647. For brainstorming to without violating the best to help carry out the
be effective which one relationship of the policies of *****the state
should be out? teacher and her c. Apply for teaching job
a. Making use of the superiors? where eligibility is not
others ideas shared a. Write an anonymous required to gain teaching
b. Teacher’s judge letter to a higher school experience before taking
mental attitude ***** official to denounce the the teachers board
c. Non-threatening superior examination
atmosphere b. Present the case d. Prepare for the
d. Openness to idea before a competent wedding she and her
authority and prepare to boyfriend have long
648. Which statement on prove the charge ***** planned to able to raise a
spaced and massed c. Call a parent-teacher family with children which
learning is correct? meeting and denounce they plan to rear as good
a. Massed learning is the superior citizen of our country
better than spaced d. Encourage the other 651. Parents are up in
arms on the telephone a. Nobody drinks at home d. Safety need
bills that pay for sex calls. except father 656. The main function of
What is the solution to b. Drug addiction has a philosophy of education
this problem? been traced to drinking is to:
a. The telephone wine ***** a. Aid the learner to build
company is to blame for c. TV ads show drinking his own personal
this is a source of fellowship philosophy
b. Parents, school and d. High taxes on liquor b. Reconsider existing
students should discuss will be deterrent to educational goals in the
this openly ***** eventual drug use light of society’s needs
c. The government 654. Cooperatives have c. Provide the academic
restriction have no teeth branched out to background prerequisite
d. Parents allow this to consumers cooperative. to learning
make their children Schools have included d. Define the goals and
modern the concepts of set the direction for which
cooperatives. Where is it education is to strive *****
652. Cooperative is practiced? 657. Which technique/s
encouraged in as many a. School book stores enable/s a teacher to
groups as possible. What b. Schools uniform identify and eventually
agency controls the purchases assists students with
different cooperatives? c. School canteen ***** interpersonal difficulties?
a. Security and Exchange d. Class stores a. Anecdotal record c.
Commission ***** Cumulative record
b. Department of Local 655. A student collapsed b. Personal inventory d.
Government in her social studies Sociogram *****
c. Commission on Audit class. It was found out 658. Teachers and
d. Bureau of Cooperative that he did not eat her students can participate
lunch. What principle is in levels of computer use.
653. Society and media shown in the situation? Give the order of
know drinking starts off a. Psychological need computer use from
drug addiction. What b. Physiological need simplest to complex?
should be discussed in ***** a. Computer competency,
schools? c. Psychosomatic computer literacy,
competency expertise should a teacher refrain assess student’s spoken
b. Computer literacy, from doing? communication skills
computer competency, a. State classroom c. Experiments in science
computer expertise ***** regulation as clearly as to assess skill in the use
c. Computer literacy, possible of scientific methods
computer expertise, b. Teacher and the class d. Artist production for
computer competency should make as many music or art subject
d. Computer competency, regulations as possible
computer expertise, ***** 664. In what period of a
computer literacy c. Enlist student aid in the child is physical growth
659. Which one is formation of classroom fastest?
considered the “Brain” of regulation a. Prenatal period
the microcomputer? d. Enforce classroom b. Early adolescence
a. CPU ***** c. Video regulations consistently *****
Screen and fairly c. Early childhood
b. Software d. Keyboard d. Prenatal and early
662. Zero standard adolescence
660. A group activity deviation means that:
wherein one group a. The students scores 665. How does fear affect
representative presents are the same ***** the voluntariness of an
the output to the bigger b. 50% of the scores act?
group rather than obtained is zero a. Makes the act
individual pupils c. More than 50% of the involuntary *****
presenting the output is score obtained is zero b. No effect at all
known as: d. Less than 50% of the c. Increases
a. Consensus decision c. scores obtained is zero voluntariness
Jury trial d. Lessens but not
b. Composite report ***** 663. Which is the least destroy voluntariness
d. Agenda authentic mode of
assessment? 666. A group of people
661. In the formulation of a. Paper-and-pencil test asserts that their culture
classroom regulations, in vocabulary ***** is superior to another.
which of the following b. Oral performance to This exemplifies:
a. Cultural gap of the development of the acceptable?
b. Ethnocentrism ***** youth. Which practice is a. Manipulate c.
c. Cultural conflict not keeping with his role Delineate
d. Norm conflict as facilitator? b. Integrate d.
a. Considers the multiple Comprehend *****
667. A test consists of a intelligences of learners 672. When is giving
graph showing the b. Humiliates praise ineffective? When
relationship between age misbehaving pupils ***** it?
and population. Follow a c. Dialogs with parents a. Uses the
series of true-false items and with other members accomplishment of peers
based on the graph. of the community as the context for
Which type of test does d. Keeps himself abreast describing a student’s
this illustrate? with educational trends present accomplishment
a. Laboratory exercise 670. Which one indicates *****
b. Interpretative ***** a teacher’s genuine b. Provides information to
c. Problem solving enthusiasm and pride in student’s about their
d. Performance teaching? competence and the
668. Which curricular a. Sticking to teaching for value of their
move served to the moment that there accomplishment
strengthen spiritual and are no better offers c. Focuses students
ethical values? b. Telling everyone that attention on her own task
a. Integration of creative he went to teaching for relevant behavior
thinking in all subject there was no other choice d. Shows spontaneity,
b. Introduction of Value then variety and other signs of
Education as a separate c. Engaging himself in credibility
subject area ***** continuing professional 673. Which statement
c. Reducing the number education ***** applies when scores
of subject areas into the d. Belittling the distribution is negatively
skill subject remuneration one gets skewed?
d. Re-introducing Science from teaching a. The mode corresponds
as all subject in Grade 1 671. In writing to a lower value
669. A teacher is a performance objective b. The median is higher
facilitator of learning and which word is not that the mode
c. The mode and median c. LET passers ***** share the products of his
are equal d. With high moral values research, he could not
d. The mean corresponds 677. What does a because of harassment
to a high value ***** skewed score distribution from all sides. Which
674. The use of the mean? teacher’s right is
process approach gives a. The scores are violated?
the student the concentrated more at one a. Right to property
opportunity to: end or the other end ***** b. Academic freedom
a. Learn to their own b. The mode, the mean *****
b. Apply the scientific and the median are equal c. Right to one’s honor
method ***** c. The mean and median d. Right to make a
c. Make use of laboratory are equal livelihood
apparatuses d. The scores are 680. Why can the
d. Learn how to learn normally distributed calculator do arithmetic?
675. A comprehension Because:
skill of higher level which 678. What is implied by a a. A computer inside the
may be inferred or negatively skewed score calculator tells it how *****
implied from reading is: distribution? b. A watch inside direct it
a. Picking out the main a. The scores are evenly c. A typewriter inside
idea distributed from the left to does it
b. Drawing conclusion the right d. A TV inside shows it
***** b. Most pupils are 681. On which
c. Nothing specific details underachieves constitutional provision is
d. Following direction c. Most of the scores are the full or partial
676. In the Preamble of high***** integration of capable
the Code of Ethics of d. Most of the scores are deaf and blind students in
Professional Teachers, low the classroom based?
which is not mention The provision on:
about teachers? 679. A teacher discovers a. Protecting and
a. Dully licensed that a product of a certain promoting the right of all
professionals bottling company brings citizen to qualify
b. Posses dignity and about damage to teeth. education *****
reputation Much as he wants to b. Providing citizenship
and vocational training to With which technique can instruction materials
adult citizen accomplish this best? d. Academic time is used
c. Academic freedom a. K-W-L technique c. wisely
d. Creating scholarship Spider web
for poor and deserving b. Venn diagram ***** d. 687. Which is the best
students Histogram reason why teacher
682. Teaching in the 685. To build a sense of begins a lesson in Math
cognitive, psychomotor pride among Filipino by checking and
and effective domains is youth what should be reviewing on the previous
based on the concept done? day’s assignment and
that the learner is a: a. Re-study our history provides practice and
a. Moral and feeling and stress on our drills?
being achievements as a a. Check if parents guide
b. Maternal and an acting people ***** their children in the
being b. Set aside the study of making of assignment
c. Thinking, feeling and local history b. Make sure that the
acting being***** c. Re-study our history students understand the
d. Spiritual and maternal from the perspective of pre-requisite skills of the
being our colonizers lesson *****
683. Both Muslim and d. Replace the study of c. Prepare the students
Christian value marriage folklores and myths with for the mastery test
but the Muslim practices technical subjects d. Make learning
polygamous marriage 686. When necessary interesting and enjoyable
while the Christian conditions are present, for students
practices monogamous the use of inductive
marriage. What is this method is preferred 688. Which is a selective
called? because: reading technique meant
a. Cultural relativism***** a. It gives the teacher at getting at important
c. Ethical relativism more time to rest facts very fast?
b. Acculturation d. b. There is greater active a. Skim reading***** c.
Enculturation participation on the part Oral reading
684. Teacher wants to of the pupils ***** b. Scanning d. Silent
compare 2 concepts. c. It needs only few reading
689. For counseling to be b. On line research experts of the students
successful which c. Manual research 694. Some students who
assumption must be d. Computer research are high in the scholastic
avoided? aptitude test have failed
a. The environment must 692. What best indicates in college. Some who are
provide assurance of the effectiveness of below the standards set
confidentiality classroom activities? for admission but who for
b. The counselor tells the a. The laughter and various reasons were
student what to do ***** enjoyment of students admitted, attained
c. The student is willing to b. The application of satisfactory standings.
participate in the process concept learned in daily This proves that:
d. The counselor must be life ***** a. Human beings are
able to relate to the c. The utilization of varied certainly predictable
student techniques and b. Admission tests are
approaches not accurate, hence
690. Which technique is d. The variety of should not be used
most appropriate when a instructional materials c. Aptitude tests do not
teacher wants a group to used measure all factors
agree on a plan of important for success*****
action? 693. The main purpose of d. Aptitude test can be
a. Composite report the compulsory study of perfectly relied on
b. Consensus decision the Constitution in 695. If the teachers
making ***** Philippine schools is to: pattern in questioning
c. Symposium a. Develop the students consists of calling on a
d. Agenda into responsible thinking student then asking the
citizens ***** question:
691. What term applies to b. Acquaint students with a. All students may be
the search for related the historical encouraged to participate
literature by computing development of the b. The student called to
access of databases of Philippine Constitution answer may be able to
discs kept in libraries? c. Prepare students for think well of his answer
a. Compact discs law making c. The rest of the class
computer research***** d. Make constitutional may just dictate the
answer individual is exposed, self for social living *****
d. The rest of the class contained experiences d. National development
may not engage 698. The tendency to requirement and reflects
themselves in thinking of imitate elders is very search based direction
the answer ***** strong in the early 701. The child cannot
696. Freud expounded childhood stage. distinguish abstracts
that there is a period Teachers should during the sensory motor
when young girls therefore be very good: of development. Which of
experience rivalry with a. Counselors c. these techniques should
their mother for their Disciplinarians a teacher apply to
father’s affection. This is b. Role models ***** d. accommodate learning?
called: Facilitators of learning a. Make use of
a. Electra complex***** 699. How is Values individualize instruction
b. Oedipus complex Education offered in the b. Explain the lesson very
c. Achilles syndrome National Secondary well
d. Cassandra syndrome Education Curriculum? c. Utilize concrete
697. Education is a a. Emphasized in objects to clarify concept
lifelong process. This Science and Technology d. Provide variety of
simply means that b. Integrated in all subject educational toys
education: areas 702. Which of these
a. May take place c. As a separate subject systems of learning
formally or informally to ***** includes ways and
enable the individual to d. Integrated with methods which are used
grow Technology and Home in preserving and building
b. May take place Economics certain within cultural
anywhere and anytime 700. The NSEC orients communities?
the individual so desires secondary education to: a. Non-formal learning
c. Is a continuous a. The teaching of the b. Multi-level learning
process of experiencing national symbols c. Cultural learning
and reorganizing b. Health values d. Indigenous learning
experiences ***** development
d. Take place in the c. The development of 703. Which of the
school where the competencies and values following statement is
true in the use of d. Intelligence is of specific responses
experiments and determined partly by pre- c. Development task is
demonstrations in natal nutrition easily identified
teaching Science: d. Experience is similar to
a. It is valuable if used in 705. In testing which of the application situation
the context of a lesson the following is referred to
that related observation as cultural bias? 708. If this need is not
to other information a. Some culture do better met, the adolescent tends
b. It should be on test than others to be critical and always
encouraged in b. Test items are more tries to find fault. This is
elementary school since familiar with some culture the need:
the concept the c. Test will show who is a. For adventure
encompass are difficult more cultured b. For recognition
for your children d. Cultured people do c. To belong
c. It is as valuable as better on tests d. For material security
teaching by lecturing 706. Which is the most 709. The way a child
d. It is less valuable than obvious and familiar way talks and walks manifest
teaching through inquiry of reporting variability? gestures that have been
and discussion a. Standard deviation learned from models he
b. Range between had been exposed. This
704. Identical twins are highest with some culture explains what influence?
more alike than fraternal c. Standard error of the a. Affective c. Social
twins. Which of the mean b. Insight d. Cognitive
following statement d. Distribution of raw
principle supported by scores 710. Audio-visual aids
this? 707. The theory of are used in classroom
a. Environment affects identical elements in teaching to __________.
both fraternal and learning holds that a. Help make learning
identical twins transfer is facilitated more permanent
b. Intelligence hinges in when the: b. All of these
physical structure a. Teacher uses different c. Help clarify important
c. Heredity has a part in teaching devices concept
determining intelligence b. Learner has a memory d. Arouse and sustain
student’s interest c. Plausibility and b. Developmentalism
711. Which of the attractiveness of the item d. Rationalism
following is the most d. Inclusion in the item 716. Which of the
important purpose for any word that must following abilities is
using achievement test? otherwise repeated in stressed by humanistic
To measure the each response education?
_________. 714. Which of these a. Learn the different
a. Quality and quantity of criteria is the most philosophies of education
previous learning important in test b. Develop man into a
b. Quality and quantity of constructions? thinking individual
previous teaching a. The stem should c. Enjoy the great works
c. Educational and contain the central of man such as the
vocational aptitude problem classics
d. Capacity for future b. Items should be d. Make man distinctly
learning congruent with the civilized, educated and
712. Which of the objectives refined
different types of test c. A table of 717. An appreciation
covers a wide variety of specification should be lesson is one that is
objectives? prepared designed to lead the
a. True-false c. d. Options should be of class to conduct and
Matching almost the same length enjoy something. Which
b. Multiple choice d. 715. Which of these of the following
Essay philosophers is reflective statements closely
713. In a multiple choice of that of Dewey’s which approximate the meaning
test, keeping the options stresses the development of the above?
brief indicates of an individual capable a. An appreciation lesson
____________. of reflective thinking should be a lesson in
a. Inclusion in the item specifically that of being values
irrelevant clues such as able to solve the problem b. Appreciation lessons
the use in the correct be faces individually or help pupils weigh and
answer collectively? clarify values
b. Non inclusion of option a. Disciplinarianism c. c. One cannot fully
that mean the same Experimentation appreciate what one does
not understand or enjoy freedom to launch the:
d. A teacher should plan individual projects a. Discovery approach
lessons that will guide c. Assign homework and b. Conceptual technique
children to appreciate check it regularly c. Integrative approach
what is beautiful d. Assigned program d. Project method
718. Which of the material for out-of-class
following is the best time hours 722. The best way the
for a teacher to set up teacher can be of the
routine activities that will 720. Which of these is appropriateness of an
contribute to effective the most important instructional materials is
classroom management? principle that a teacher to:
a. As soon as the should follow in initiating a. Try it out before using
students have a program with positive it in class
established reinforcement? b. Consider its technical
b. Daily at the start of the a. Make sure the reward quality
session comes immediately after c. Consider its
c. During his homeroom the appropriate behavior availability
days b. Punish negative d. Consider its cost
d. On the every first day behavior and reward
of school positive behavior 723. Tasks analysis
c. Provide regular involves the breaking
719. In large classes opportunity for socially down of a learning task
where little of the work acceptable behavior into subtasks or sub
pupils can be d. Consider peer skills. Given a task to
individualized, the most approval and recognition retell a story, which of the
effective and practical following skills is not
ways to individualize 721. The trend of needed?
instruction is to: focusing attention on the a. To disseminate
a. Devise group activities child’s interests, abilities information
which afford every pupils and needs and on the b. To outline a selection
an opportunity to work at improvement of c. To identify topic
his own community living sentences
b. Give the pupils necessitate the use of d. To arranged events in
sequence blackboard? learners and bright
learners to organize and
724. You are assigned to 726. The new teacher generalize is for teacher:
teach students with entered a noisy a. To make the bright
varied abilities. You want classroom. She shouted learners guide the dull
to teach a more immediately at the ones in learning to
homogenous grouping. students desperately generalize
Which type of grouping trying to get order and b. To make the bright
will tend to benefit your discipline. Since then the learners to generalize
students? teacher has not and the dull ones to
a. Mixed ability grouping controlled the class. memorize
b. Low ability group Which is the most c. To give the dull
c. Within class ability probable cause of the learners to more concrete
grouping teacher’s failure? experiences to serve as
d. High ability grouping a. The students reaction basis for generalizing
to the teacher is the d. To give both the dull
725. Which of the consequence of her and bright learners
following examples behavior concrete and abstract
illustrate the use of b. Rules are not defined experiences to serve as
questions to focus pupils and procedures to basis for generalizing
attention on the key sustain order is not put
points of the lesson? into place 728. Which of the
a. What is Rizal Park c. The new teacher following will do the first
known for? wants to show the class to establish good class
b. Why are machine who is authority management?
made goods cheaper d. The class wants to test a. Discuss the required
than those made by the ability and patience of rules for proper class
hand? the teacher behavior
c. Have you ever b. Discuss the work plan
enjoyed watching the 727. The educational for the year
clouds on a bright day? implementation of c. Prepare a seat plan
d. Who came while I was research findings relative d. Train the class in the
writing on the to the ability of dull distribution of material
competent person new social situation when
729. A student was b. The psychological test given encouragement
diagnosed to have a high result are still valid and and support. How can
IQ but is failing in his reliable this be done?
academic subject. What c. When the records are a. By discovering his new
should the teacher do to updated interest
help him? d. When the records are b. By giving him room
a. Talk to his parents kept for ready reference responsibility
b. Examine his study when needed c. By assigning “peers or
habits 732. Which of the Buddies” to him
c. Talk the student and following is a major d. By giving him special
find out his problem advantage in using help
d. Refer him to the arithmetic mean? 735. Who expounded on
guidance counselor a. It is more commonly the need to study the
used than other child carefully for
730. Which is the true measures individualized instruction?
foundation of the social b. It is simple to compute a. Da Feltre c. Boccacio
order? c. It discriminates b. Erasmus d. Ascham
a. Strong, political between the lowest and 736. Which of the
leadership the lowest following should a
b. The reciprocation of d. It is more than stable teacher do if she cannot
rights and duties than the median pay the monthly
c. Equitable distribution 733. When I am engaged installment of an
of wealth in an external criticism in appliance she got from a
d. Obedient citizenry a historical research, department store in their
731. When do test, what am I occupied with? town?
inventories and career The _______ of the a. Reject any notice of
information become document. demand for payment to
effective for counseling a. Author c. Source make the impression that
services? b. Authenticity d. she did not receive
a. When the data Accuracy b. Move to another
generated are interpreted 734. Learners often find it neighborhood to escape
on time by professionally much easier to fit into a payment
c. Inform the manager of a. Minimize the d. A teacher paid on an
the store personally and unemployment problem hour basis, entertain her
make a satisfactory b. Produce globally students with stories until
arrangement of payment competitive graduates the end of the period
on or before the due date c. Protect the rights of 741. You have a pupil
of payment the citizen who is so talkative,
d. Offer the return of the d. Ensure the teaching of naughty and aggressive
used appliance to the Filipino that he is a burden to the
store on the condition 739. Teacher should bear entire members of the
that she will be refunded in mind that the period of class. How would you
on the monthly greatest mental remedy this problem?
installment she paid development is from: a. Talk to him seriously
737. Which of the a. 9 to 12 years c. 6 to 9 b. Call the parents for
following will you years dialogue
recommended to a senior b. 12 to 15 years d. 3 to c. Report the case to the
high school scholar who 6 years principal
is impregnated by a 740. Which of the d. Reprimand him always
fellow student? following is the best 742. What should a
a. Tell her parent about situation wherein you can teacher do before
her condition balance responsibility constructing items for a
b. Stop schooling until and accountability? particular test?
after she gives birth a. A teacher paid on an a. Prepare a table
c. Direct her to an hour basis, takes her time specifications
abortion clinic with the subject matter b. Review the previous
d. Force her boyfriend to until the end of the period lessons
marry her b. A teacher paid on an c. Determine the length
738. The government hour basis, teaches as of time for answering it
prescribes a higher much as she could for the d. Announce to students
percentage on the duration of the period the scope of the test
administration of c. A teacher paid on an 743. Under which of the
educational institution to hour basis, spends most multiple choice type of
Filipino citizens in order of the time on the latest test can this question be
to: gossips in showbiz classified? “Which of the
following statement done by gravitational situation wherein you can
expresses this concept in force is a learning task. balance rights and
different forms?” At what level of cognition authority?
a. Association c. is it? a. Allow all their only
Difference a. Application c. daughter’s suitor to come
b. Definition d. Cause Evaluation and go as she pleases
744. Of the following b. Knowledge d. b. Caution their only
types of test which is the Comprehension daughter’s choice of a
most subjective in 748. Setting up criteria for boyfriend
scoring? scoring test is meant to c. Censor all their only
a. Matching type c. increase their ________. daughter’s suitor
Multiple choice a. Objectively c. Validity d. Choose a life-partner
b. Simple recall d. Essay b. Reliability d. Usability for their only daughter
745. In which of these 749. Which of the 751. Classical
research methods can following you will do to an conditioning theory is
the researcher control examinee you caught always attributed to him
certain variable? cheating and who offered for his experiment
a. Experimental c. a certain sum of money involving the dog’s
Descriptive to keep quiet? salivation as a reaction to
b. Ex post facto d. a. Confiscate his test the sound of the buzzer.
Historical paper and report him to a. Skinner c. Bandura
746. During the first the examination b. Lewin d. Pavlov
grading period, a student supervisor 752. The singing of
obtained failing marks in b. Motion him to keep national anthem is an
five academic subjects. quiet and watch for him offshoot of the
Which of the following after the examination philosophical ideas of:
tests would best explain c. Announce to all a. Naturalism c.
his performance? examinees the name of Socialism
a. Mental ability c. the cheater b. Nationalism d.
Attitude d. Ignore him but let him Pragmatism
b. Personality d. feel you saw him 753. An adolescent
Aptitude 750. Which of the combines his ability to
747. Measuring the work following is the best use deductive and
inductive reasoning in David and his classmates d. Inter quartile range
realistic rules that he can at once remark: “What’s 758. A child who is cold
respect and live by. wrong?” what does this towards that people
When he does this, how mean? among him might have
does he perceive his a. Guidance counselor failed to attain what basic
environment? are perceived to be goal based on Erickson’s
a. He sees the world “almighty and omniscient” theory on psychological
through the eyes of the b. The parents of student development?
people Jay must be of the a. Autonomy c. Initiative
b. He interprets events delinquent type b. Trust d. Mistrust
from a limited point of c. Reporting to a 759. Under the learning
view Guidance Office is often to do, which of the
c. He sees events apart associated with following instruments
for himself and other misbehavior must be acquired so that
people d. Student Jay is a a person can perform his
d. He views the world “problem” student work effectively?
from his own perspective 756. Which of the a. Competence c.
754. Which of the following assessment Compromise
following statement is the techniques best assess b. Insights d.
main reason why should the objective “ plans and Communication
negative words be designs an experiment to 760. What do you think
avoided in constructing be performed” would be the actions of a
multiple choices tests? a. Rating scale c. teacher who found out
a. Might be overlooked Checklist and has proven that his
b. Stems tends to be b. Paper and pencil test principal is involved in the
longer d. Essay malversation of funds of
c. More difficult to 757. What type of their school?
construct option measure of variation a. Malign him trough an
d. Increase the difficulty easily affected by the anonymous letter
of the test item extreme scores? b. Present the charge to
755. Student David was a. Quartile deviation a complete authority
asked to report to the b. Standard deviation c. Ignore what the
guidance office. Student c. Range teacher has discovered
about this action of the AVOIDED in constructing sorts of inedible
principal true or false test? substances. This is
d. Circulate this issue and I. Verbal clues and called:
let it become a gossip specific determiner a. Pica c. Encopresis
761. Standard deviation II. Terms denoting b. Enuresis d. Anorexia
is to measure of definite degree of amount 766. What is the main
variations as ______ is to III. Taking elements advantage of using table
measure of central directly from the book of specification when
tendency. IV. Keep true and false constructing periodic
a. Quartile deviations c. statement the same in test?
Mean deviation length a. It reduces the scoring
b. Range d. Mode a. I and III only c. I, II, time
762. Which statement and IV b. It improves the
is/are true in constructing b. I, II and III d. II and IV sampling of content areas
matching type of test? only c. It makes test
I. The option and 764. The discriminating construction earlier
descriptions not index number 15 is 0.44 d. It increase the
necessarily homogenous this means that: reliability of the test result
II. Description in Column a. Equal number of 767. If a teacher is an
A and options in Column student got the correct advocate of banking
B answer concept in Education he
III.The options must be b. More students from the or she viewed student
greater than the upper group got the item as?
description correctly a. Clear account to be
IV. The directions must c. More students from the filled up by the teacher
state the basis of lower group got the item b. Dormant account to be
matching correctly activated by the teacher
a. I, II, and III c. I, II, and d. The test item is very c. Wobble account to be
IV easy balanced by the teacher
b. II, III, and IV d. I, II, III, 765. Some children go d. All of the above
and IV through a period of 768. Teacher Maechelle
763. Which of the intense appetite when is a neophyte teacher.
following should be they eat or chew on all One time a mother of one
of her students b. Guides students in as authorities and models
confronted and maligned learning process by children at the early
her in front of her c. Shares responsibility in childhood stage. What
colleagues. How should counseling does this statement
conduct Teacher Myla d. Inspires student to imply?
react on this kind of interesting lessons a. Parent Teacher
situation? 770. Which of the conference should
a. Walk away and ignore following would best always be an activity in
the mother describe the role of the school
b. Answer back the schools? b. Parents should enforce
mother and malign her a. To educate the citizens strict discipline at home
too b. To educate the citizens and teachers in school
c. Wait until the emotion to change the society c. Teachers and Parents
of the mother subsides c. To fit the citizens into should serve as role
and invite her to discuss the society models at all times
the concern with the d. All of the above d. Teachers should
principal or guidance 771. Manual aesthetic demand complete
counselor activities involving obedience from the
d. Allow the mother to attitudes and feelings are learners in school
keep on maligning her primarily expressive of 773. Which of these
until it’s her turn to do the emotions and values not statements regarding
same thoughts. An example of professional’s teachers is
769. Ms. Teodora is this motor skills is: the major difference in
always guarded by the a. Baking a cake with the professionalization of
principle that she has a background music teachers and teaching as
foremost responsibility as b. Saving a board for the promulgated in
a teacher. Given the wall of a book cabinet Presidential Decree 1006
following which do you c. Dancing and playing and in Republic Act
think is the main musical instruments 7836?
responsibility of Ms. d. Manipulating a bowling a. Assigned at the tertiary
Sanchez? ball to achieve a strike level in both private and
a. Ask the challenging 772. Parents and state colleges and
question teachers are considered universities
b. Assigned at the provided the present Exams for teacher
elementary and philosophy of education 777. A number of
secondary levels in both was the contribution of researchers found the
public and private the Aquino effects of maternal
schools administration. Which of employment on children’s
c. Holder of valid this statement is NOT achievement are:
professional license and consistent with our a. Fully establish c.
certificate of registration education philosophy? Positive and negative
d. Appointed on full-time a. Restore Ethical and b. Negative d. Hardly
basis and on permanent moral values establish
status b. Appreciate the roles of 778. Babyhood is often
774. Social stratification foreigners on the referred to as a “critical
is greatly developed in historical saga of the period” in the
the classroom. Which of country development of
these activities would be c. Foster nationalism and personality because:
an effective way of patriotism a. Changes in the
avoiding or minimizing d. Introduce vocational personality pattern take
this? and scientific efficiency place
a. Encourage higher 776. In the preamble of b. At this time the
education aspirations the Code of Ethics for foundations are laid upon
among the less privileged Professional Teachers, which the adult
pupils which of the following personality structure we
b. Provide limited teacher descriptions is built
experience to children of included in the Code of c. The brain grows and
less privileged classes Ethics of the Professional develops at such an
c. Assign leadership roles Teachers? accelerated rate during
to the children of the a. With satisfactory babyhood
upper social class teaching performance d. At the time the baby is
d. Avail the influence of b. Duly licensed exposed to many hazards
mass media for children professional both physical and
of all classes c. Persons of dignity and psychological
775. The freedom reputation 779. Research
constitution which d. Passed the Licensure established that complete
coordination of motor cooperatively engaged c. New Secondary
activities is attained at: with the teacher in a Education Curriculum
a. Childhood stage c. group project the children d. National Elementary
Pre-natal stage will discipline themselves Achievement Test
b. Infanc d. as each member of the 786. In a classroom it is
Adolescence stage group exercises: possible to see the
780. Which of these a. Special interest teacher doing the
theories holds that b. Moral compulsion following to faci9litate
human activity is based c. Obedience to the learning.
on the interaction of teacher I. The class reads a
stimuli and responses? d. Peer influence workbook on the
a. Vector c. Association 784. Operation “Return to characteristics of animals
b. Social learning d. the basic” was launched II. The class copies the
Cognitive field by the department of characteristics of animals
781. It is the process by education, Culture and from books
which an organism sports not only to III. The class goes out to
inherent the upgrade pupil the zoo to observe the
characteristics traits of achievement but also to: animals
the patients: a. Emphasize the IV. The teacher shows
a. Fertilization c. importance of the 3Rs posters of animals
Maturation b. Develop thinking skills Which of these teacher’s
b. Heredity d. c. Encourage pupils to activities reflects an
Development study interactive environment?
782. When the daughter d. Serve as the basis for a. III and IV c. I and IV
is completing with the learning continuum b. I and II d. III only
father for the fathers 785. Which of the 787. As provider for the
attention, the daughter is following embodies the education act of 1982,
said to be experiencing: operation “return to the how much are the
a. Sexual deviation c. Basics”? institutions of learning
Identity crisis a. National Secondary encouraged to set higher
b. Electra complex d. Achievement Test standards of quality over
Oedipus complex b. New Elementary and above the minimum
783. If children are School Curriculum required for state
recognition? c. Emphasize for administration of all
a. Formal education creativity and productivity educational institutions
b. Academic freedom d. Integrated with shall be vested in the
c. Voluntary accreditation technology and home citizens of the Philippines
d. Continuing economics is stipulated in:
Professional Education 791. Which of the a. P.D.1006 c. 1987
(CPE) following is a mark of a Constitution
788. The following is good teacher? b. P.D. 6-A d. P.D. 176
TRUE in the development a. Has the habit of 794. During the class
of understanding in early preparing for visual aids reunion of teacher, Eric
childhood except: b. Has the mastery of the learned that most of his
a. Sensory experiences lesson classmates are
b. Abilities to reason & to c. Has the control of the successful in their fields.
see relationship class Also, he found out that
c. Ability to ask questions d. Has the capability to most of them are wealthy
d. Ability to explore their implement corporal because they have
environment punishment chosen a lucrative
789. The first 792. The main function of profession. Confronted
kindergarten also known a philosophy of education with this situation, how
as “a garden where is to: should teacher Eric
children could grow” was a. Aid the learner to build react?
the product of research his own personal a. Hide to those asked
by: philosophy his real profession
a. Froebel c. Pestalozzi b. Define the goals and b. Tell with pride that he
b. Herbart d. Rousseau set the direction for which is a teacher by profession
790. Values development education is to strive c. Leave the event so as
is integrated in all c. Provide the academic to avoid being asked
subjects in the NSEC background prerequisite about his profession
while values education is: to learning d. Answer not their
a. Emphasized in science d. Reconsider existing question concerning his
and technology education goals in the profession
b. Offered as a separate light of society’s needs 795. Teachers being the
subject 793. The control and the trustee of the cultural and
educational heritage of her. What she did was to was due for his teacher-
the nation are under write and secretly writer
obligation to: distribute copies of c. He had the modular
a. Promote obedience to anonymous letter to her lessons published when
the laws of the state colleagues. What should they worth publishing
b. Promote national pride, have been done instead? d. He wants to exclusive
cultivate love of country & a. Secretly give beneficiary of the royalty
instill allegiance to the anonymous letter only to from the modules
constitution the people concerned 799. Teacher Vincent, a
c. Transmit to learners b. If the charge is valid; teacher for thirty two
such heritage and elevate present such charge years, refuses to attend
national morality under oath before her seminars. He claims that
d. All of the above school head his thirty two years of
796. The principal is very c. Ask a third party to teaching is more than all
much interested in a write the anonymous the seminars he is asked
quality professional letter to prevent her from to attend. Are his
development program of being involved actuation and thinking in
her teachers. Which of d. Talk to the married accordance with the code
the following should she man with whom Ms. Del of Ethics for professional
consider to realize this? Mundo is having an illicit teachers?
a. Prescribe by top affair a. Yes, provided he has
educational teachers 798. The principal ask his an excellent performance
b. Required for renewal good teacher to write rating
of professional license modular lesson in b. No, non attendance to
c. Responsive to Filipino, then he had seminars would mean no
identified teachers needs them published with his increase in salary
d. Dependent on the name printed as author. c. No, a professional
availability of funds Which is unethical in this teacher, regardless of
797. Ms. Soriano wants case? teaching experience,
to help in ending Ms. Del a. He burdened the ought to go through CPE
Mundo’s act of immorality teachers with work not d. Yes, because he
but doesn’t have to related to teaching taught for thirty two years
encourage confronting b. He got the merit which and may have mastered
the traded lawyer does he uphold?
800. Education is a b. Develop students with a. Positivism c.
continuous process of into responsible thinking Progressivism
experiencing and visiting citizen b. Essentialism d.
or reorganizing c. Acquaint students with Existentialism
experiences according to the historical 805. Teacher Paul is an
a Progressivist. What development of the inspiration to almost all of
does it mean? constitution the students. His
a. Education takes place d. Make student’s efficiency and
anytime and anywhere constitutional expert effectiveness in the
b. Education goes on 803. Which of the profession is truly
throughout life following situations which outstanding. Which of the
c. Education happens violate the principle of following describes this
formally or informally respect? attitude towards him?
d. Education begins and a. Teacher B is giving a. Naturalism c. Idealism
ends in school special favor to students b. Progressivism d.
801. The main to please so that she can Perennialism
contribution of the Arroyo get a remarkable result in 806. Teacher finds
Administration in the evaluation teaching in a multi
education is Republic Act b. Teacher A tells her cultural classes very
No. 9155. This provision: student that what teacher challenging. Which
a. Started the national B taught is wrong among the following
scholarship program c. Teacher B, upon choices will alleviate the
b. Renamed the DECS to learning what teacher A difficulty of addressing
DepEd did, asked the students these challenges?
c. Conceptualized the K- not to attend her class a. She must embody a
12 Education Program d. All of the above curriculum rather than
d. Established the study 804. If a teacher states perspective
now-pay later system that specialization knows b. She must nurture
802. The main Purpose more and more about diversity rather than
of compulsory education less and less, hence it is practicing domination and
of the constitution is to: better to be generalist. oppression
a. Prepare students to be What kind of philosophy c. She must consider
stereo typing rather than b. Available d. Affordable 812. Which of the
cultural identities and 809. When asked, computer-based
biased attitudes students of teacher Eric instructional tool can help
d. She must welcome described him as you revise written work
one sided view rather someone who knows such as short stories and
than the recognition of what he is talking about. essays?
biases Teacher Eric therefore a. Spreadsheets c.
807. Ms. Janina is the exhibits a power known Desktop publishin
most admired pre-school as: b. Database d. Word
teacher in her school. a. Expert power c. processing
Which among the Referent power 813. Ms. Agatep wants to
following can best explain b. Reward power d. show to her class a
her being a good Legitimate power magnified picture of the
teacher? 810. When choosing an Mt. Pinatubo’s crater
a. She endorses all the instructional aide or fixed on a bond paper.
projects of the school for device, the primary Which of the following
her students consideration of the tools can she use?
b. She manages to instill teacher would be: a. Slides c. Overhead
control to her students a. Suitability c. Projector
c. She gives easy exams Availability b. Filmstrip d. Opaque
to her students b. Cost d. Efficiency Projector
d. She adheres to the 811. To show disapproval 814. As a teacher
want of the parents for to the misbehavior of the employing the project-
their children student, Teacher Paul based multimedia
808. Teacher Vincent clears his throat and learning (PBML) strategy,
bought a hamster in the looks intently at the what are some limitations
class during the lecture erring. This classroom teachers expect from the
about mammals. The management style is encounter?
hamster is a device commonly known as: I. There is a need for
commonly known as a a. Direct appeal c. extending the to use
REALIA. Teacher can Dropping of name several media
bring realia only when: b. Proximity control d. II. The presentation of the
a. Workable c. Feasible Signal interference product is not an easy
task of child’s interest in his learning skills?
III. The technology skills body? a. Act on impulse and
to produce a product a. Comparing themselves cannot concentrate
maybe lacking with others b. Must take stimulants
a. I, II, III c. I only b. Looking at themselves which shorten attention
b. III only d. II only in the mirror span
815. B.F. skinner is a c. Commenting on c. Are given sedatives
known psychologist and various parts o the body which make them listen
the one who first to d. Looking the picture of d. Are mildly retarded
describe operant adult men and women 821. Many concerned
conditioning. Which of the 818. Motor development parents commonly make
techniques is an is satisfied by a particular the mistake of:
application of operant child who; a. Deliberately creating
conditioning? a. Recognize the different high levels of stress for
a. Master learning sizes of toys given to him their child
b. Process approach b. Learns how to walk, b. Unintentionally
c. Project method run, steer and jump rewarding their children
d. Computer assisted c. Has playmates within for creating stress
instruction the neighborhood & is c. Attempting to protect
816. Which of the popular among kids their children from all
following choices is d. Known how to control stress
considered as social his emotions because he d. Unintentionally
force that affects the could not ride the motor creating high levels of
school and the cycle stress for their child
curriculum? 819. Which among the 822. Stuttering is
a. Nature of knowledge following drugs is commonly caused by:
b. Learners’ commonly used for a. Problem with physical
characteristics children with ADHD? origin
c. Learner’s style a. Valium c. Retalin b. Psychological
d. Changes in gender b. Haldol d. Thorazine consequence of
roles 820. What is the main permissive parenting
817. Which of the reason why children with c. Side effects of
following is an expression ADHD have limited authoritarian parenting
d. Result of fixation Dyslexia individual differences
caused by conflicts b. Childhood autism d. c. Reward as a potential
during toilet training ADHD means of increasing the
823. How can parents 826. When a person’s participation
foster initiative and moral choices are d. Allowing the student to
independence in determined by the direct be exposed to various
children? consequences of actions. teaching techniques
a. Mastering He is most like in the 829. All of the following
psychomotor skills stage of: shows respect for
b. Encouragement from a. Conventional c. Post individual differences
parents when a child conventional except:
plans carries out a task b. Concrete d. Pre a. Give greater attention
c. Identity versus role conventional to gifted learners
confusion 827. What is the b. Treat all learners alike
d. None of the above motivation of the person while the classroom
824. Which stage who paints for the sheer c. Provide for a variety of
considers teachers, peers enjoyment of creating learning activities
and adults outside the artwork? d. Prepare modules for
home important in a. Insufficient c. slow learners in class
shaping attitude toward Extrinsic 830. Students who are
oneself? b. Intrinsic d. Intrinsic disobedient and display
a. Initiative versus guilt & Extrinsic negative attitudes
b. Industry versus 828. Providing variety of towards others are best
inferiority learning activities to handled by teacher who
c. Trust versus mistrust students is a will:
d. Integrity versus despair characteristics of a a. Detain him after office
825. When a child teacher who understands hours for him to do what
manifest mutism, self- the principle of: he has been ordered to
destructive behavior and a. Proactive teaching as b. Avoid giving him
echolalia, the child might a modem technique of orders or if you do and
be showing the teaching the objects take back the
symptoms of: b. Facilitating learning order
a. Anorexia nervosa c. with emphasis on c. Take every opportunity
to praise him for every not anymore able to settings?
positive attitude displays absorb additional a. Kohlberg c.
d. Insist on compliance to information. This Montessori
the same degree required phenomenon is known b. Piaget d. Froebel
of pupils as: 837. The concept that
831. Which of the a. Stagnation c. learning to read or write
following develops critical Boredom does not happen quickly
thinking skills among the b. Procrastination d. but is build upon many
students? Plateau of learning small steps that occur
a. Asking convergent 834. Planned ignoring, over the course of the
question signal interference and child’s early childhood.
b. Blind obedience to proximity control are a. Innate literacy
authority techniques used in: b. Emergent literacy
c. A willingness to a. Operant conditioning c. Functional literacy
suspend judgment b. Managing surface d. Academic Literacy
d. Asking low level behavior 838. A boy is closer to his
questions c. Managing temper mother and a girl is close
832. A child who always tantrums to her father. These
fights with his/her d. Life space interviewing instances are under:
classmates, who has a 835. A foreigner who is a. Latent stage c. Phallic
very short attention span, studying here in the stage
and who has frequent Philippines was turned off b. Oedipal Complex d.
tantrums is believed to be by the Filipinos way of Pre-genital stage
suffering from: eating Balut and Frogs: 839. Laughing at two
a. Mental retardation a. Xenocentrism c. year old child who uttered
b. Attention deficit Colonial mentality a bad word is not a
hyperactivity disorder b. Ethnocentrismd d. proper thing to do
c. Down syndrome Culture shock because in this stage of
d. Learning disability 836. Who among the the child’s life, the child
833. Teacher Elaine has following claimed that is:
been lecturing for more children are natural a. Considering the views
than an hour and she learners and therefore of others
notice that students are must be taught in natural b. Distinguishing right
from wrong d. Uses pro-active 845. The nearest to the
c. Distinguishing sex discipline method real thing according to
differences 842. Which of these Edgar Dale’s Cone of
d. Socializing philosophies is reflective Experience is:
840. The school director of the Dewey’s which a. Watching demo
emphasizes the necessity s\tresses the b. Viewing images
of clean and green development of an c. Attending Exhibit
environment to contribute individual capable of d. Hearing and listening
to effective teaching and reflecting thinking 846. This is pre-planned
learning. This is an specially that of being collection of sample of
example of: able to solve the problem student works, assessed
a. Establish rapport he faces individually or results and other output
between teachers and collectively? produced by the
pupils a. Disciplinism c. students:
b. Providing an Experimentation a. Anecdotal report c.
atmosphere conductive to b. Developmentalism d. Portfolio
learning Rationalism b. Diary d. Observation
c. Providing adequate 843. The present military image
physical facilities training in our school 847. Which of the
d. Utilize educational curriculum is an influence following statements is
technology of: one of the strength of an
841. The teachers are a. Sparta c. Greece autobiography as a
facilitators of learning. b. Rome d. Athens technique for personality
Which of the following 844. Basic education appraisal?
negates this principle? includes secondary a. It can replace data
a. Performs not only education. Which of the obtain from other data
classrooms roles but following contributed to techniques
community involvement the establishment of b. It may be read by
as well secondary schools? unauthorized people
b. Focus background of a. Reformation c. c. It gives complete data
the students Realistic about the author
c. Familiar with the latest b. Rationalism d. Human d. It makes possible
innovations in teaching Education presentation of intimate
experiences b. Discuss the unsettled advice can you give him?
848. The present bill of the students a. Report every student’s
Philippine Teachers c. Discuss the progress misbehavior to the
professionalization Act as well as the principal
had its beginning on what deficiencies of the b. Agree with the class on
period of roman history? stu7dent what rules to follow
a. 295 B.C. - schools d. Discuss the complaints c. Set the ground rules
were elementary only of other teachers and for the whole class
b. A.D. 100 - A.D. 175 - classmates of the d. None of the above
government increased its students 853. Which of the
subsidy for education 851. After listening to the following is a result of
c. 132 B.C. - A.D. 100 - homily of the priest about compressing a file?
Latin literature and fidelity, Catherine has a a. The file size is smaller
grammar were perfected moment of reflection. Her b. The file deleted
d. A.D. 274-259 - understanding of the c. The three character
government establish a value of fidelity has extension is hidden
monopoly on education become deeper as she d. All file properties
849. Whose philosophy related this to her past 854. Mrs. Anita Kusing
influences the present experience. This typifies was not accepted by a
emphasis on character what kind of philosophy? certain company because
education and values a. Constructivism c. of her age. This
education in our school Humanism discrimination based on
system? b. Reconstructivism d. age is called
a. Tagore c. Confucius Existentialism a. Gerontism c. Agelism
b. Gandhi d. Bonifacio 852. Teacher Vincent is a b. Autism d. Senilism
850. During the new teacher. He realizes 855. Vincent parents do
distribution of the report that handling his not want their child with
card, which of the student’s misbehavior is ADHD to undergo drug
following must be the a very demanding aspect treatment, their better
foremost concern of of classroom alternative would be
teacher? management. In this a. Behavior modification
a. Discuss the projects of regard he thought of or behavior management
the school giving up teaching. What b. Psychotherapy
c. Punishment intelligence a. Achievement test
d. Progressive inhibition 859. Which of the b. Extended essay test
856. Ripple effect can following is an example of c. Completion test
also be seen in norm-referenced d. Objective test
misdemeanor. The interpretation? 862. A student finding it
teacher should therefore: a. Josh’s test score is hard to read. When the
a. Reinforce positive higher that 89% of the guidance counselor
behavior class traced the child’s history,
b. Immediately responds b. Francis set up his the counselor was able to
to misbehavior laboratory equipment in 2 find out the student came
c. Be consistent in minutes from a dysfunctional
classroom management c. RJ solve five problems family, aside from that the
d. Count 1 to 10 before correctly out of thirty child was abused and
she gets angry words neglected. What could
857. Metacognition is d. Bea must spell twenty have caused the students
primarily characterized five words correctly out of reading disability?
by: thirty words a. Poor teaching
a. Recalling the past 860. A type of error b. Emotional factors
lesson committed in grading the c. Neurological factors
b. Thinking about their performance of the d. Immunity
thinking students by the rater who 863. The best example
c. Visualizing in advance avoids both extremes of Operant conditioning
d. Formulating hypothesis the scale and tends to among the following is:
858. A person who is rate everyone as revenge a. Fostering conductive
friendly and has a a. Severity error learning environment
capacity to make people b. Central tendency error b. Connecting facts and
laugh possesses: c. Generosity error concept
a. Interpersonal d. Logical error c. Using reinforcement
intelligence 861. Which of the d. Using manipulative
b. Naturalistic following test items can device
intelligence be effectively measure 864. The failure in the
c. Spatial intelligence higher order of cognitive test of independence
d. Intrapersonal learning objectives? among Filipino students
can be attributed to of understanding in early a. Child centered design
a. Lack of skills childhood, except: b. Life situation design
b. High degree of a. Abilities to reason and c. Humanistic centered
dependence to authority to see relationship design
c. Strong family ties b. Sensory experiences d. Experience centered
d. All of the above c. Ability to explore their design
865. Mrs. Soriano is environment 871. Which of the
admitted for being an d. Ability to ask questions following statements
effective classroom 868. Teacher Eric would about computer viruses
manager. She is not only like to compare and are TRUE?
friendly but at the same contrast plant cell vs. a. Text files are the only
time be_______ animal cell. He would files to be corrupted by
a. Confident c. Analytical most likely use: virus.
b. Business-like d. a. Fishbone diagram c. b. Files damaged by
Buddy-buddy Tree Diagram computer viruses can be
866. Which of the b. Venn Diagram d. cured
following would be the KWL chart c. Files are always
most fitting action for the 869. Which of the permanently damage by
teacher who is having a following are the basic viruses
relationship with his/her components of curriculum d. Compressed files can
student? design? never be damaged by
a. Deny the relationship a. Assessment, teaching viruses
b. Continue the strategies and textbooks 872. The students of
relationship and exercise b. Content, structure and Teacher Corazon feels
utmost professional deliver that their teacher has an
discretion about this c. Philosophy, goals and “eye behind her head”
c. Enjoy the relationship objectives this characteristics of the
while it last d. Standards, learning teacher is known as:
d. Defer the relationship competencies, and a. Referent Power c.
until they are already to evaluation Pygmalion Effect
admit it 870. The following are b. Withitness d.
867. The following is examples of learning Rosenthal Effect
TRUE in the development centered design except: 873. After studying the
Principle of identity, d. Art over science transportation of the said
teaching Beng ask her 876. Clearance has poet. Which of the
students to determine inherent skills in taking following can she use so
which among the given care of plants. It is highly she won’t need to spend
set of problems conforms possible that she has much?
to the said identity. This ________ intelligence. a. Distant communication
shows that teacher beng a. Intrapersonal b. Instant messaging
upholds this kind of intelligence c. Video conferencing
philosophy? b. Naturalistic d. Podcasting
a. Perennialism c. intelligence 880. Teacher Lally wants
Essentialism c. Spatial intelligence her students to express
b. Progressivism d. d. Existential intelligence their opinions regarding a
Naturalism 877. According to certain government issue.
874. How will you classify Erickson, a child who is Which of the following
the purpose of the school cold towards that people can she use to do this?
as it concerns with the among him might have a. Forum c. Email
training and preparation failed to attain what basic b. Blog d. Group
of citizens for the world of goal based on messaging system
work? psychosocial 881. Which is the first
a. Political Purpose c. development? step in planning periodic
Economic Purpose a. Autonomy c. Initiative test?
b. Cognitive Purpose d. b. Trust d. Mistrust a. Select the type of test
Social Purpose 878. What is most likely to be used
875. A child submitted a characteristic of children b. Construct a table
poor written report but aged 3 to 5 according to specification
package with boringly Erickson? c. Go back to the
colored paper cover. This a. Mischievous c. Lazy instructional objectives
showcase____. b. Egocentric d. Altruistic d. Determine the group of
a. Art over academics 879. Mrs. Cruz dreams to whom the test is intended
b. “porma” over organize a seminar with a 882. Below is a list of
substance known poet from another methods used to
c. Substance over country but she cannot establish the reliability of
“porma” afford to spend for a a test, which method is
questionable due to networking site classrooms?
practice and familiarity? 885. Teacher Vincent is I. Motivational
a. Split half c. Kuder assigned to handle a II. Unique instructional
Richardson multiple grade class. capabilities
b. Equivalent form d. Test What instructional III. Increase Teachers’
retest material must be used to Productivity
883. What is meaning of provide the need of each a. I only c. II and III only
TOS in the parlance of grade? b. I and II only d. I, II,
test construction? a. Differentiated materials and III
a. Table of specifics to cater to different levels 888. Using Bloom’s
b. Table of specification b. Multisensory materials Taxonomy, the highest
c. Table of specific test c. Colorful, useful and among the following is:
d. Table of subject durable materials a. Critical
884. Ms. Soriano will be d. Materials of high level Comprehension
absent for two days thinking skills b. Critical evaluation
because of a national 886. A document c. Integration
conference. She wants publishes by a school d. Literal comprehension
her student to work on a district that identifies 889. Among the following
certain module. What is rules of behavior that educators, who proposed
the fastest way of must be followed by the placement of children
sending the module to anyone using the school in a “prepared
her students while she is districts computers, environment”?
away? network, or internet a. Thorndike c. Kilpatrick
a. Share it to all her connection. b. Montessori d. Froebel
students in her live a. Ethical internet use 890. To improve
account policy comprehension and
b. Email a module to her b. Classroom rules retention among the
students c. Acceptable use policy students, the teachers
c. Send the module d. Fair use best option would be use
through group instant 887. Which of the a. Six thing hats
messaging following are the b. SQ3R
d. Message each of the rationales behind using c. Contextual Clues
students on a social technology in the d. Autodidactic Activities
891. Who among the dedication for the children he accomplish?
philosophers considered 894. To increase the I. Administer a parallel
habits and reasons as difficulty of a multiple test
equally important Forces choice test item, which of II. Split the test
to be cultivated in the following should be III. Construct a variety of
education? done? items
a. Avicenna c. Jerome a. Make the options equal IV. Administer the same
Bruner in length test twice
b. Aristotle d. Maria b. Make the options a. I, III, IV c. I, II
Montessori homogeneous b. I, II, IV d. I, IV
892. What practice(s) will c. Make it grammatically 897. Teacher Vilma
demonstrate the teachers correct constructed a matching
genuine concern on the d. Make the stem type test item. IN her
learning of students? 895. Teacher Anna set column of descriptions
a. Confer progress of 85% accuracy in a test on are combinations
students to their parent predicting the kind of presidents, current
b. Guides students to weather given 5 different issues, and sports. Which
meet their learning goals atmospheric conditions. of the constructing a
c. Validates if learning May obtain a score of matching type test items
goals were met 82% can be interpreted was NOT followed?
d. All of the above as: a. The descriptions must
893. Which of the a. She is 3% short of the be homogenous
following is the most set percentile score b. The options must be
important component of b. She did not meet the greater than the
educational reform? set standards by 3% descriptions
a. Allowing the students c. She obtained 82% c. The descriptions must
to participate more in percentile score be heterogeneous
extra-curricular activities d. She is higher than 82% d. Arrange the options
b. Implementing a better of the group according the orders
curriculum for students 896. Teacher Vincent 898. Which of the
c. Hiring only the wants to establish the following does not belong
excellent teachers reality of test in biology. to the group when we talk
d. Involving parents in the Which of the following will about projective
personality test?
professional-education-
900_2.html
a. Sentence completion
test
b. Word association test
c. Interview
d. Thematic Apperception
test
899. Which of the
following is the main
purpose of administering
a pre test and post test to
the stu8dents?
a. Measure the value of
the materials used
b. Accustom the students Comments
in frequent testing
c. Measure gain in 1.
learning
wilimnsonherrryFebruary
d. Measure the 10, 2020 at 8:19 PM
effectiveness of
Mesmerized article written
instruction on this blog with other
900. Which is implied by relevant information. It is
straight to the point that
a positively skewed score how we can improve our
distribution? skills as well as how we
a. The mean, The median can be represented to a
new stream of
and the moderate are professionalism.
equal Online Gastroenterology
quiz
b. Most of the scores are
low REPLY
c. Most of the scores are
Post a Com
high
d. The mode is high https://professionalteacher-
reviewer.blogspot.com/201
9/09/let-reviewer-

Das könnte Ihnen auch gefallen